Atomic Structure

You might also like

Download as pdf or txt
Download as pdf or txt
You are on page 1of 49

Join our telegram channel: @Jee_neet_topper

Chapter
2
Atomic
Structure

Key Concepts
Rutherford Model ________________________
DALTON’S THEORY OF ATOM 1. The atom of an element consist of a small positively
charged nucleus which is situated at the centre of the
Atom was considered as a hard, dense and smallest indivis- atom and carries almost the entire mass of the atom.
ible particle of matter. It cannot be destroyed or created. 2. An electron rotates arround the nucleus. Since very few
a-particles (1 out of 20,000) are returned back it means
Drawbacks ______________________________ the size of the nucleus is very small and is very rigid.
3. The order of diameter of an atom is 10–10 m or 1 Å and
The discovery of various sub. atomic particle e, p, n, x-ray. of the nucleus is 10–15 m = 1 fm.
4. rn = (1.33 ¥ 10–15) A1/3 m. A = mass number rn =
MODELS OF ATOM radius of Nucleus of any element having mass number A.
Failure of Rutherford Model
Thomson’s Model ________________________ 1. According to electromagnetic theory, when a charged
Atom is electrically neutral. It contains positive as well as particle moves with acceleration, they emit energy in
negative charges. According to Thomson, atom is a uniform the form of electromagnetic radiation. According to
sphere of positive charge with electron embedded in it. This Rutherford model electrons are revolving around nu-
was known as plum pudding modle at that time. cleus and emit energy in the form of
electromagnetic radiation. As a re-
sult of which electron loses energy
Rutherford’s Scattering Experiments ________
continuously and should collapse
on nucleus following a spherical
path. Rutherford model failed to
ZnS explain the stability of atom.
2. If electrons loose energy continuously the observed
Au plate spectrum may be continuous but it will consist of well-
defined line of definite frequency. Hence the loss of
energy by the electron is not continuous in an atom.
Result
Calculation of distance between a particle and centre of
1. Most of the a-particle passed through the foil. nucleus:
2. Some of them were deflected through small angles. Total energy at initial point
3. Very few (1 in 20,000) were deflected at large angles
1
(180°). Ti = K.E. + P.E. = mv2 + 0 ...(i)
2

IIT JEE PC-V1_02.indd 1 3/21/2017 5:14:49 PM


Join our telegram channel: @Jee_neet_topper

2.2 Physical Chemistry-I for JEE (Main & Advanced)

Total energy at final point Types of Spectrum________________________


K ◊ q1q2 1. Emmision spectrum: When any substance absorbs energy
Tf = 0 + ...(ii)
r or radiation, its atoms get excited to higher energy state. After
some time when they return to their original state, they emit
radiation and spectrum. This is called emission spectrum.
i f
M
fi fi Emission
r spectrum
E E
By energy conservation law tio
n
Ti = Tf dia
Ra
q2 = charge on Nucleus of an element
q1 = charge and particle = 2 ¥ 1.6 ¥ 10–19 C 2. Absorption spectrum: When radiation is passed through
m = mass of µ particle = 4 ¥ 1.6 ¥ 10–27 Kg a substance, certain wavelength absorbed and when this
1 2 K ◊ q1q2 outcoming radiation is analysed at the photographic plate we
mn = observe dark line at plane of observed wave length. This is
2 r
2 K ◊ q1q2 called absorption spectrum.
r= It may be continuous, band or line.
mn 2
2 ¥ 9 ¥ 109 ¥ (2 ¥ 1.6 ¥ 10-19 )( Z ¥ 1.6 ¥ 10-19 )
=
4 ¥ 1.6 ¥ 10-27 ◊ n 2 Continous Band Line
Wave: It is a disturbance in medium A A
through which transfer of energy takes
B B
C C
place from one place to other without a D
E
D
E
F
the transfer of matter.
F
G G
Spectrum H H
Parameters: before I I

(i) Wavelength: Distance covered l experiment Absorption spectrum B C E F Spectrum


by the wave in completion of one after
experiment
cycle. Radiation Emitted
(ii) Frequency (n): The number of cycles completed in E1 Vapour radiation
E2
1 sec.
(iii) Time period: Time taken by the wave in one cycle Quantum theory of radiation: According to Max Plank,
1 radiation emitted from hot bodies, radiant energy (light or
completion where T =
n heat) is not emitted continuously but discontinuously in the
1 form of small packets called Quanta. It means atoms or mol-
(iv) Wave No. n =
l ecules emit or absorb the energy in the form of Quanta [E =
Electromagnetic waves do Electric field hn]. They absorb or emit energy as hn, 2hn ... not in the form
not require any medium to of 1/2 hn, 3/2hn, etc., i.e., fraction of Quanta is not possible.
propagate from one place to Wave direction E = hn [h = Plank constant = 6.63 ¥ 10–34 J.S.]
another place. It has two com- and TE = n (hn) n is the number of Quanta of particular
ponents: (i) Electric field and Magnetic field frequency.
(ii) Magnetic field. They are l (in Å) 0.01 1.0 150 3800 7600 6 ¥ 106 3 ¥ 109
perpendicular to one another and to the direction of propaga- Cosmic
g rays x rays uv rays visible Infra Micro Radia
tion. Light is an electromagnetic wave. rays
light red wave wave
When white light is passed through prism it get dispersed
Frequence
into different colours. It is called spectrum of white light. in (Hz)
1017 1057 1014 1011 109

t Bohr’s atomic model


eligh
Whit
R
O
Y
B
G m Ze
I e–
e r
V

IIT JEE PC-V1_02.indd 2 3/21/2017 5:14:51 PM


Join our telegram channel: @Jee_neet_topper

Atomic Structure 2.3

1. According to Bohr’s atomic model, e– rotate around


h2 n2 2 n2h2
the nucleus in a circular orbit. The required centripetal r= ¥ and n =
force is provided by electrostatic force of attraction be- 4p 2 mKe2 Z 4p 2 m 2 r 2
tween the electron and nucleus. [6.63 ¥ 10-34 ]2
=
mn 2 Ke ¥ Ze 4 ¥ (3.14) 2 ¥ 9.1 ¥ 10-31 ¥ 9 ¥ 109 ¥ (1.6 ¥ 10-19 ) 2
=
r r2 n2
¥ m/sec.
2. Electrons revolve in those orbit which have angular z
nh
momentum equal to (where n is an integer 1, 2, n2
3,...) 2p r = 0.529 Å
Z
nh Ê n 2 ˆ nh
mnr = mn r = mn ¥ Á 0.529 A∞˜ =
2p Ë z ¯ 2p
3. Each allowed orbit have definite energy. These are
Z
called stationary orbit. Electrons when revolve in v = 2.18 ¥ 106 m/sec.
stationary orbit do not radiate energy. The energy of n
stationary orbit increases as we move farther from the Total energy = K.E. + P.E.
nucleus. KZe 2 1
4. Electrons emits energy only when it transitions from P.E. = - and K.E. = mn 2
r 2
higher energy states to lower energy states.
mn 2 KZe 2
We know = 2
r r
1 2 1 È KZe 2 ˘
\ K.E. = mn = Í
2 2 Î r ˙˚
Humphry series 6
5 1 È KZe 2 ˘ È KZe 2 ˘
Pfund n1 = 6 \ T.E. = K.E. + P.E. = Í ˙ + Í- ˙
d n2 = 7, 8 … 2Î r ˚ Î r ˚
Bracket
r are 4
Inf 3
Paschan
Infra Red 1 KZe 2
Infra Red
n1 = 3 2 T.E. = -
2 r
Balmer
Visible n2 …
1
Lyman n1 = 2
U.V n2 = 3, 4 … 1 9 ¥ 109 ¥ (1.6 ¥ 10-19 ) 2 Z
n1 = 1 =-
n2 = 2, 3 … 2 n2
0.529 ¥ 10-10
Z
First line a Line [minimum energy line]
1 Ê z2 ˆ
È1 1˘ T.E. = -13.6 Á 2 ˜ ev/atom
= Rz 2 Í 2 - 2 ˙ R = 1.097 ¥ 107 Ën ¯
l n
Î 1 n2˚

Second line b line. Ê z2 ˆ


T.E. = - 313.6 Á 2 ˜ Kcal/mole
Calculation of radius, energy and velocity of e– in nth Ën ¯
Bohr atom
Electrostatic force of attraction provide required centrip- 1 eV = 1.6 ¥ 10–19 joule
etal force Result Species having only one electron system around nu-
cleus in any orbit
K ( Ze)e mn 2 ...(i)
F= = (i) Total Energy = –Kinetic Energy of electron
r2 r (ii) Potential Energy = 2 Total Energy
nh (iii) Ionisation Energy = Kinetic energy = –Total Energy
mn r = ...(ii)
2p
Drawbacks of Bohr’s Model________________
n2h2
\ m 2n 2 r 2 = 1. It explains the spectrum of species having only one
4p 2 electron H atom, He+ ion and Li++ ion. It does not ex-
È n2h2 ˘ plain the spectral of atoms having more than one elec-
\ r=Í 2 2˙ tron.
Î 4p m ¥ KZe ˚

IIT JEE PC-V1_02.indd 3 3/21/2017 5:14:53 PM


Join our telegram channel: @Jee_neet_topper

2.4 Physical Chemistry-I for JEE (Main & Advanced)

2. When a resolving power spectroscope is used it was È 2p r ˘


observed that the spectral line in H spectrum is not a \ Time taken = Í second
single line but a collection of several lines which are Î V ˙˚
very close to one another. Bohr’s theory does not ex- De Broglie Equation ______________________
plain the fine spectrum of H atom.
According to this theory, small particles like electrons,
3. It does not explain the splitting of spectral lines into a
protons, neutron, a particle, etc., behave as particle nature as
group of finite lines under the influence of magnetic
well as wave nature. The wavelength of particle is given by
field [Zeman effect] and electric field [Stark effect].
Jusitification of quantisation angular momentum. h h
l= =
4. Spectrum of isotope of H was expected to be same by mn P(momentum)
Bohr’s theory but it is different by experiment.
Ionisation energy: Minimum amount of energy which is Proof of Bohr’s Theory by De broglie Relationship
required to eject the electron from the atom from its ground When electrons rotate around nucleus in any orbit, the
state. number of completed wave is equal to the number of orbit.
E• = 0 It means No. of waves = No. of oribtal
\ IE = E• – En = 0 – [En] È h ˘
From nl = 2p r or n Í ˙ = 2p r 1
IE = - En Î mn ˚ 2
n=1 1l = 2p r
Total number of spectral line that can be obtained when nh
electron in nth energy state gets transitioned. n=2 2l = 2p r mn r = 5
2p 3
( Dn)( Dn + 1) proved.
= 4
2 n=5 5l = 2p r
n Some relations:
4 1. Between Momentum (P) and Kinetic Energy (K.E.)
2. Between K.E. and Wave length (l)
3 h h h
l= = =
P 2 KE.m 2q ¥ V ¥ m
for Electron
2
6.625 ¥ 10-34 150
l= = Å
2 ¥ 1.6 ¥ 10 -19
¥ 9.1 ¥ 10 -31
¥V V

1 Since
1 2
K.E. = mn fi 2K.E. = mn2
hc z 2 È 13.6 z 2 ˘ hc 2
DE = En2 - En1 = = 13.6 2 - Í- ˙=
l n2 Î n12 ˚ l fi 2K.E. ◊ m = (mn)2 = (P)
È1 1 ˘ hc 1 13.6 È 1 1˘ \ P = 2KE ◊ m
= 13.6 en Z 2 Í 2 - 2 ˙ = fi = Í n 2 - n 2 ˙ en
n
Î 1 n2˚ l l hc Î 1 2˚ 3. Relation between K.E. and mass of particle for same
1 È1 1˘ value of de broglie wavelength.
= Rh Z 2 Í 2 - 2 ˙ RH = 1.097 ¥ 107 m -1
l n
Î 1 n 2˚
h h2
We know l = \ l2 =
Rh = Rydberg constant 2(KE).m 2KE.m
\ for same l (Wave length)
Frequency of revolution:
Distance travelled in 1 revolution = 2p r 1
r \ KE µ
meter. m
Distance travelled in 2p r meter h 1 mass of e– < Proton < Neutron < a particle
revolution \ for same de-Brogile Wave length order of kinetic
1 energy
\ Distance travelled in 1 metre h revolution e– > Proton > Neutron > a particle
2p r
V
\ Distance travelled in V meter h = Frequency
2p r

IIT JEE PC-V1_02.indd 4 3/21/2017 5:14:55 PM


Join our telegram channel: @Jee_neet_topper

Atomic Structure 2.5

Heisenberg Uncertainity Principle __________ Note:


It is impossible to measure both the position and velocity (or 1. Current depends on intensity. i µ I (intensity)
momentum) of a microscopic particle simultaneously with 2. K.E. depends upon frequency and K.E. µ frequency.
absolute accuracy.
According to this theory, product of uncertainity in posi-
tion and uncertainity in momentum is always greater than or Stopping potential: It is the minimum required potential to
h stop moving electrons.
equal .
4p 1
h K.E. = mn2 = hn – W
(Dp) (Dx) ≥ 2
4p = Applied voltage ¥ charge [q ¥ V]
h
Dx ¥ m ◊ DV = V = stopping potential
4p
h I2
Dx ¥ DV =
4p m I1
I2 > I 1
Photoelectric effect: Hertz Light = hv
(1887) discovered that when Current
a beam of light of suitable
wavelength falls on a clean e–
metal plate (such as cesium)
in vacuum, electrons are
Vaccum Voltage
emitted from the surface
of the metal plate. This Stopping potential is same.
phenomenon involving G
the emission of electrons
from the surface of a metal by the action of light is known QUANTUM NUMBER
as photoelectric effect. The electrons so emitted are called
photoelectrons. The set of four integers required to define an electron com-
Photon energy = K.E. of e– + threshold energy. pletely in an atom are called quantum number.
K.E. of e– = photon energy – threshold energy. 1. Principle quantum number: It was proposed by Bohr.
It is called orbit, shell or energy level.
1 2 È1 1 ˘ It is denoted by n. value of n is positive integer. It is
mn = hn - hn 0 = hc Í - ˙ = hn - f
2 Î l l0 ˚ denoted by KLMNOP.........
It shows the average distance from the nucleus
n0 = Threshold frequency or critical frequency.
n2
l0 = Threshold wavelength or critical wavelength. r = 0.529 Å.
z
It show the average energy of electron
Experimental Result
1. If n < n0 fi Photoelectric effect does not occur. What z2
E = –13.6 eV.
ever intensity is high. n2
2. If n > n0 fi Photoelectric effect. If intensity Increases num- Maximum number of electron in any orbit is given by
ber of ejected electron Increases and K.E. remain same. 2n2
3. If n Increases then K.E. of ejected e Increases since It has maximum number of subshell = n
K.E. = hn – hn0 Maximum number of orbital = n2.
We know K.E. = hn – hn0 ∫ y = mx – C. 2. Azimuthal quantum number:
It was proposed by Sommer Field.
It is donated by l.
It shows the shape of orbital.
KE Value of l for any value of n 0, 1, 2, 3 ...(n – 1)
l = 0 fi s subshell
tan q = h = slope l = 1 fi p subshell
v0 q l = 2 fi d subshell
l = 3 fi f subshell
v0
v It has maximum no. of electron In any subshell = 2(2l
–hn0 + 1)

IIT JEE PC-V1_02.indd 5 3/21/2017 5:14:57 PM


Join our telegram channel: @Jee_neet_topper

2.6 Physical Chemistry-I for JEE (Main & Advanced)

Maximum no. of orbital In any subshell = (2l + 1)


l Naming l subshell shape ELECTRONIC RULE
1 0 s 0 s spherical
2 0, 1 sp 1 p dumble Pauli rule: According to this rule, 2 electron in an atom cannot
3 0, 1, 2 spd 2 d double dumble have same set for all four quantum numbers. In other words,
we can say an orbital cannot have more than two electron.
4 0, 1, 2, 3 spdf 3 f complex
Hund’s rule of maximum multiplicity: Electrons are
The value of Azimuthal angular momentum distributed among the orbital of the subshell in such a way
h as to give the maximum number of unpaired electron with
= l (l + 1) = orbital angular moment a parallel spin. Pairing of electrons in the orbitals belonging
2p
3. Magnetic quantum number: to the same subshell does not take place until each orbital
It shows the orientation of electron around nucleus. belonging to that subshell has got one electron each.
It is represented by ‘m’. 3P4  
The value of m for any value of l is [–l...0...+ l]
Total number of orbital in any subshell = (2l + 1) 3P3  
n=4
Auf bau principle: The electrons are added progresively to
l=0 1 2 3 various subshell in the order of increasing energies starting
with the subshell of the lowest energy.
m=0 –1 0 +1 –2 –1 0 +1+2 –3 –2 –1 0 +1 +2 +3 Or
the order of ascending energy of subshell can be calculated
Spin quantum number:
from (n + l).
It shows the rotation of electron on its self axis.
Lower the value of (n + l) for an subshell having lower
1 1 energy. If two subshell have same the value of (n + l) then
It has value + and - where +ve sign shows a clockwise
2 2 the subshell which has low value of ‘n’ will have low energy.
and –ve sign shows anticlockwise rotation.
Orbital n l n+l
h
Spin angular moment = s ( s + 1)
2p 1s 1 0 1
2s 2 0 2
SHAPE OF ORBITAL
2p 2 1 3
3s 3 0 3
s-orbital
3p 3 1 4
py py 3d 3 2 5
4s 4 0 4
p-orbital px px
pz 4p 4 1 5

z 4d 4 2 6
y z
+
4f 4 3 7
x y x 5s 5 0 5
+
5p 5 1 6
dxy dxz
d-orbital dz2
z y Multiplicity: 2|S| + 1
S = total spin.
2 3
x x for ns np
1 1 1 3
d(x2 – y)2 S= + + =
dxz 2 2 2 2

IIT JEE PC-V1_02.indd 6 3/21/2017 5:14:59 PM


Join our telegram channel: @Jee_neet_topper

Atomic Structure 2.7

3 Probability curve: It was found that wave function y can be


Multiplicity = 2 +1= 4 expressed as a product of two functions.
2
y[r ◊ qf] = R(r) ◊ f(qf)
1s Ø Ø
2s 2p radial angular
3s 3p 3d function function
4p wave function of different Hydrogen orbital
4s 4d 4f
2
5s 5p 4d 5f for 1s = R(r) = 3/2
e–s
a
6s 6p 6d
1
7s 7p for 2s = R(r) = (2 – s) e–s/2
2 2 a03/2

1
for 2p = R(r) = se–s/2
Exchange energy = It is the number of possible transition of 2 6 a03/2
electron in the same subshell.
e–x = Hyperbola curve

fi 4 h2
when a0 = = 0.529 Å
4p mke 2
2

r
fi 3 s=
a0
= 10
R = wave function
fi 2
r = distance from nucleus
R2 = intensity of wave function
fi 1 Angular node = l
Radial node = n – l – 1
Node and nodal plane: It is the space where probability of
finding electron is zero. The plane where probability of find- Total node in a shell
ing electron is zero is called nodal plane. (n – l – 1) + l = n – 1

Schrödinger Wave Equation _______________ Radial Probability Density _________________


This model is based on particle and wave nature of electrons The square of the radial wave function for an orbital gives the
known as wave mechanical model of an atom. radial probability density of finding an electron at a point at
distance ‘r’ per unit volume.
∂2 x ∂2 y ∂2 z 8p 2 m
+ + + 2 ( E - V )y = 0
∂x 2 ∂y 2 ∂z 2 h
Radial Probability Distributed
h = Plank constant, E = Potential electron Function (4p r2R2) ________________________
V = Potential energy, m = Mass of electron The radial probability density at a radial distance ‘r’ is R2.
y = Wave function Therefore, radial probability of finding the electron in a vol-
Significance of y: The wave function may be regarded as the ume dn will be R2dn.
amplitude of vibration expressed in terms of coordinate x, y, Radial probability is the probability of finding electron in
z. The wave function may have positive or negative values a radial shell between spheres of radius ‘r’ and ‘r + dr’ where
depending on the values of coordinate. dr is small radial distance volume (dn) of spherical shape of
Significance of y2: For electromagnetic radiation, the square thickness ‘dr’.
of amplitude is proportional to the intensity of light. y2 at any r + dr
point is proportional to the probability of finding of electron.
N r dr
Orbital: Region or space where probability of finding
electron is maximum (90 – 95%).

IIT JEE PC-V1_02.indd 7 3/21/2017 5:15:01 PM


Join our telegram channel: @Jee_neet_topper

2.8 Physical Chemistry-I for JEE (Main & Advanced)

4 4 Total probability distribution = R2 ◊ dn = R2(4p r2dr)


dn = p (r + dr)3 – p r3
3 3 = (4p r2) dr = radical distribution function
4 Zeffective = Z – s
= p [(r + dr)3 – r3]
3 s can be calculated by Slater Rule
= p [(r3 + dr3 + 3r2dr + 3rdr2 – r3] (i) s for 1s or 1s2 = 0.30
Ø Ø (ii) s for ns and ns np electron = [Valance electron – 1] ¥
0.35 + [(n – 1) Shells electron] ¥ 0.85 + [Rest electron
neglect neglect from (n – 1) shell and nucleus] ¥ 1
4 (iii) s for d/f electron = [given d/f electron – 1] ¥ 0.35 +
dn = p [3r2dr], dn = 4p r2dr
3 [Rest electron from given d/f and nucleus] ¥ 1

R10 R2, 0 R3, 0 R2, 1

r r r
r
for (1s) (2s) (3s) (2p)

2 2
R1,0 2
R2,0 2
R3,0 R2,1

r r r r

2 2 2
4pr2R1,0 4pr2R2,0 4pr2R3,0

rmax rmax rmax

(1s) (2s) (3s)

2 2 2
4pr2R2,1 4pr2R3,1 4pr2R3,2

rmax rmax rmax


r

(2p) (3p) (3d)


Comparision between 3s, 3p and 3d.

2 2 2
4pr2R3,0 4pr2R3,1 4pr2R3,2

rmax rmax rmax


r

(3s) (3p) (3d)

IIT JEE PC-V1_02.indd 8 3/21/2017 5:15:03 PM


Join our telegram channel: @Jee_neet_topper

Atomic Structure 2.9

Solved Examples

1. Find the ratio of frequencies of violet light (l = 4.10 ¥ 1242


10–5 cm) to that of red light (l = 6.56 ¥ 10–5 cm). Also DE (ev) = = DE (ev)
l (nm)
determine the ratio of energies carried by them.
hc 1242
Sol. We know E = = hn T.E. = eV = 24.84 eV and I.E. = 13.6 eV
l 50(nm)
hc K.E. = T.E. – I.E. = 24.84 – 13.6 = 11.24 eV
C = vl and E = = hn
l 6. Find the longest wavelength of the radiation which is
nv lR E n l required to pickout the electron from the first excited
= = 1:1.6 V = V = R = 1:1.6 = 0.625 : 1 state of Paschan series?
n R lv ER n R lV
Sol. First excited state of Paschan series means n = 4
2. A 100 W power source emits green light at a wave- È1˘ hc
length l = 5000 A. How many photons per minute are \ E4 = –13.6 Í 2 ˙ = –0.85 =
Î4 ˚ l
emitted by the source?
hc 1242
T.E. = P ¥ t = 100 ¥ 60 E• = 0 \ DE = 0.85 = \l= nm
Energy emitted by source (T.E.) = power ¥ time l 0.85
= Number of photon ¥ Energy of one photon 7. The frequency of series limit of Lyman series is n1 and
series, limit of Paschan series is n2, and frequency of
hc 6.63 ¥ 10-34 ¥ 3 ¥ 108 emission of electronic energy from 3 to 1 is n3. Find the
Energy of one photon = =
l 5 ¥ 103 ¥ 10-10 relation between n1, n2 and n3.
\ Number of photons emitted •
T.E. 100 ¥ 60 ¥ 5 ¥ 103 ¥ 10 –10
= =
Energy of one photon 6.63 ¥ 10 –34 ¥ 3 ¥ 108
= 1.5 ¥ 1022 n2
3. Find the ionisation energy of H, He+, Li++.
n1
ÈZ2 ˘ E3 Paschan
Sol. EH = –13.6 Í 2 ˙ ev/atom
În ˚ E2 Balmer
È1˘ n3
E1 = –13.6 Í ˙ = –13.6 eV
Î1˚ E1 Lyman
E• = 0
\ DE = E• – E1 = 13.6 eV for hydrogen atom Sol. E• – E1 = hn1 E• – E3 = hn2
E3 – E1 = hn3
È 13.6[2]2 ˘ + From the figure
= 0 – Í- ˙ = 13.6 ¥ 4 eV for He
Î 1 ˚ hn3 = hn1 – hn2
È13.6[3]2 ˘ n3 = n1 – n2
++ 8. Find the magnitude of the current flowing in fourth
=0– Í ˙ = 13.6 ¥ 9 eV for Li
Î 1 ˚ Bohr orbit in hydrogen atom in one revolution.
4. Find the wavelength of the radiation which is required q e eV
for the very first transition of H atom? Sol. i = = =
T 2p r 2p r
hc
Sol. E2 – E1 = very first transition means n(2) to n(1) V
l È1˘
1 È 13.6 ¥ 1˘ hc hc We know V = 2.18 ¥ 106 Í ˙ and r = 0.529 ¥ 10–10
-13.6 ¥ 2 - Í- = = 10.2 eV = În˚
2 Î 12 ˙
˚ l l [n 2 ]
,n=4
Z
10.2 ¥ 1.6 ¥ 10 -19 1˘
\ l= metre È
6.63 ¥ 10 -34 ¥ 3 ¥ 108 [1.6 ¥ 10-19 ] ¥ Í2.18 ¥ 106 ¥ ˙
\ i= Î 4 ˚ amp.
5. A radiation of wavelength 50 nm incident on H atom -10 È 1 ˘
0.529 ¥ 10 Í 2 ˙
in ground state. Find the kinetic energy of ejected elec- Î4 ˚
tron.
9. A particle having K.E. 5 eV collides with the atom at
hc 6.63 ¥ 10-34 ¥ 3 ¥ 108 ground state. Predict the collision: Elastic or non-elas-
Sol. DE ( joul) = =
l l (meter) tic?

IIT JEE PC-V1_02.indd 9 3/21/2017 5:15:05 PM


Join our telegram channel: @Jee_neet_topper

2.10 Physical Chemistry-I for JEE (Main & Advanced)

Sol. We know minimum energy –13.6/4 eV For charge Particle


required would be 10.2 eV 10.2 eV KE = Charge ¥ Voltage
for the transition since it is –13.6 eV For electron
only 5 eV. So collission will
6.625 ¥ 10-34
be perfectly elastic. l=
10. A hydrogen-like species is observed to emit six radia- 2 ¥ V ¥ 1.6 ¥ 10-19 ¥ 9.1 ¥ 10-31
tion originating from all possible transition between a
12.27 ¥ 10-10
group of level. These levels have energy between –0.85 = meter
eV to –0.544 eV eV including both level. Find (a) V
Quantum numbers and (b) Atomic number. 12.27 150
= Ū Å
z 2 V V
Sol. En = –13.6 = – 0.85 eV
n2 14. Calculate de broglie wavelength of an electron acceler-
6 radiation means Dn = 3 ated through a potential difference of
z2 n+3 (a) 75 volt (b) 300 volt
E(n+3) = –13.6
(n + 3) 2 n+2
150
= –0.54 Sol. (a) l = Å = 1.414 Å
n+1 V
En (n + 3) 2
=
E( n + 3) ( n) 2 n 150
(b) l = Å = 0.7 Å
n+3 85 300
\ = = 1.25, n = 12
n 54 15. Calculate the uncertainity of position of electron if it is
equal to uncertainity of distance travelled by electron
Z2 in 1 sec.
\ En = –13.6 = – 0.85 Z can be calculated
n2 Sol. Dx = DV (Heisen berg)
11. Calculate the wavelength of photo electron which is h
ejected from ground state of hydrogen atom by absorb- \ (Dx) ◊ (Dx) =
4p m
ing 14 eV photon?
Sol. K.E. = T.E. – I.E. h
\ Dx =
here DE = K.E. = 14 – 13.6 = 0.4 eV 4p m
e– eject from atom if given Total Energy (T.E.) > Ioni- 16. If the velocity of electron in third orbit of He+ ion is ‘x’,
sation Energy (I.E.) calculate the velocity of electron in second orbit of Li+2
6.625 ¥ 10-34 ion.
\ l=
Z
2[0.4 ¥ 1.6 ¥ 10-19 ).9.1 ¥ 10-31 Sol. We know v = 2.18 ¥ 106 m/sec
n
12. Arrange in increasing order K.E. of electron, protons For He+ Z = 2 and n = 3
neutron and a particle.
2
Sol. When All have the same de broglie wavelength. 2.18 ¥ 106 ¥ = x …(i)
\ for same l (Wave length) 3
For He+ z = 2 and n = 3
1 3
\ KE µ 2.18 ¥ 106 ¥ = V …(ii)
m 2
1 Equation (ii)/(i)
K.E. µ for same wavelength
m V 3 3 9
\ = ¥ =
Since order of mass me < mp < mn < ma x 2 2 4
\ Order of Kinetic Energy a < n < P < e 9
v= x
13. Show that de broglie wavelength of electron accelera- 4
1 17. Calculate K.E. and P.E. of electron in third orbit of H
Ê 150 ˆ 2 atom.
tion through V volt is nearly given by l = Á A.
Ë V ˜¯ Sol. We know T.E. = –K.E. and P.E. = 2 ¥ Total Energy
h
Sol. We know for l =
p z 2 -13.6 ¥ 1 -13.6
and T.E. = –13.6 = = = –1.51 eV
h h n2 32 9
\ l= =
2(KE)m 2(q.V )m \ K.E. = 1.51 eV and P.E. = 2 ¥ 1.51 = –3.02 eV

IIT JEE PC-V1_02.indd 10 3/21/2017 5:15:07 PM


Join our telegram channel: @Jee_neet_topper

Atomic Structure 2.11

18. Suppose the potential energy between electron and pro- (Electrostatic force of attraction) Provide centripetal
- Ke 2 force
ton at a particular distance r is given by . Use
3r 3 mn 2 K K
Bohr’s theory to obtain energy of such a hypothetical = \ v=
r r m
atom.
Sol. d(U) = F ¥ dr K = mn2
È - Ke 2 ˘ We know
dÍ 3 ˙
dU - Ke 2 È dr -3 ˘ Ke 2 nh
F= = Î 3r ˚ = Í ˙= ¥ 3r -4 mn r =
dr dr 3 Î dr ˚ 3 2p
F = Ke2r–4 = electrostatic force of attraction nh
n=
mn 2 2p mr
and we know F = (from Bohr’s theory)
r K nh
Ke 2 mn 2 =
= ...(i) m 2p mr
r4 r
nh m
1 2 r=
\ K.E. = mn 2p m K
2
1 Ke 2 nh
KE = r=
2 r3 2p Km

nh n2h2 T.E. = K.E. + P.E.


We know mnr = fi m2n2r2 =
2p 4p 2 1 2
2 2 = mn + K ln r
n h 2
fi m(mn2)r2 =
4p 2 1
From (1) = K + K ln r
2
È Ke 2 ˘ n 2 h 2 4p 2 Ke2 m
m Í 3 ˙ r2 = \ È1 ˘
Î r ˚ 4p 2 n2h2 = K Í + ln r ˙
Î2 ˚
1 Ke2 È Ke2 ˘ Ke2
T.E. = K.E. + P.E. = + Í ˙ T.E. = È1 nh ˘
2 r 3 Î 3r 3 ˚ 6r 3 T.E. = K Í + ln
Î 2 2p Km ˙˚
Ke2
= 3 1
È 4p 2 mKe2 ˘ 20. If the ratio of K.E. of ejected photoelectron is when
6Í ˙ K
Î n2h2 ˚ photo frequency n1 and n2 given. The calculate thresh-
n6 h6 old frequency of metal?
T.E. =
384p 6 K 2e4 m3 We know K.E. = h(v – v0)
19. Suppose P.E. between electron and proton at a separa- Kinetic Energy (K.E.1) = h(v1 – v0) v2 – v0 = K[v1 – v0]
tion ‘r’ is given by u = K ln r. K is a constant. For such a Kinetic Energy (K.E.2) = h(v2 – v0) v2 – v0 = Kv1 – Kv0
hypothetical atom, calculate the radius of nth Bohr orbit
and energy. K.E.1 1 n - n0
= = 1 v0(K – 1) = Kv1 – v2
Sol. u = K ln r K.E.2 K n 2 - n 0
du d [ K ln r ]
F= =
dr dr [kn1 - n 2 ]
\ n0 =
d [ln r ] K (k - 1)
F= = =F
dr r

IIT JEE PC-V1_02.indd 11 3/21/2017 5:15:09 PM


Join our telegram channel: @Jee_neet_topper

2.12 Physical Chemistry-I for JEE (Main & Advanced)

Exercise
LEVEL I 1 1
(a) 5, 0, 0, + (b) 5, 1, 0, +
2 2
1. The energy of hydrogen atom in its ground state is 1 1
(c) 5, 1, 1, + (d) 6, 0, 0, +
–13.6 eV. The energy of the level corresponding to 2 2
n=5 15. The orbital diagram in which the Aufbau’s principle is
(a) –0.54 eV (b) –5.40 eV violated is
(c) –0.85 eV (d) –2.72 eV 2 s 2 px 2 p y 2 p 2 s 2 px 2 px 2 px
2. The spectrum of He+ is expected to be similar to that (a) ≠Ø ≠Ø ≠ z
(b) ≠ ≠Ø ≠Ø ≠Ø
of -
2 s 2 px 2 px 2 px 2s 2 p 2 p 2 p
(a) Li+ (b) He (c) H (d) Na (c) (d) ≠Ø x x z
≠Ø ≠ ≠ ≠ ≠Ø ≠Ø ≠
3. What possibly can be the ratio of the de Broglie wave-
length for two electrons having the same initial energy 16. The total number of neutrons in dipositive zinc ions
and accelerated through 50 volts and 200 volts? with mass number 70 is
(a) 3 : 10 (b) 10 : 3 (c) 1 : 2 (d) 2 : 1 (a) 34 (b) 40 (c) 36 (d) 38
4. The uncertainty in the momentum of an electron is 17. Which of the following sets of quantum numbers repre-
1.0 ¥ 10–5 kg m s–1. The uncertainty of its position will sents an impossible arrangement
be (h = 6.626 ¥ 10–34 kg m2 s–1) n l m s
(a) 1.05 ¥ 10–28 m (b) 1.05 ¥ 10–26 m 1
–30 (a) 3 2 –2
(c) 5.27 ¥ 10 m (d) 5.25 ¥ 10–28 m 2
5. The orbital with zero angular momentum is 1
(a) s (b) p (c) d (d) f (b) 4 0 0
2
6. The electronic configuration of the Mn4+ ion is
(a) 3d44s0 (b) 3d24s1 (c) 3d14s2 (d) 3d34s0 1
(c) 3 2 –3
7. Which of the following ions has the maximum number 2
of unpaired d-electrons? 1
(d) 5 3 0
(a) Zn2+ (b) Fe2+ (c) Ni3+ (d) Cu+ 2
7
8. The total spin resulting from a d configuration is 18. The explanation for the presence of three unpaired
(a) 1 (b) 2 (c) 5/2 (d) 3/2 electrons in the nitrogen atom can be given by
9. Given: K L M N (a) Pauli’s exclusions principle
2 8 11 2 (b) Hund’s rule
The number of electrons present in l = 2 is (c) Aufbau’s principle
(a) 3 (b) 6 (c) 5 (d) 4 (d) Uncertainty principle
10. The value l and m for the last electron in the Cl– ion are 19. The electronic configuration of an element is 1s2 2s2
(a) 1 and 2 (b) 2 and +1 2p6 3s2 3p6 3d5 4s1. This represents its
(c) 3 and –1 (d) 1 and –1 (a) Excited state
11. The maximum energy is present in any electron at (b) Ground state or cationic form
(a) Nucleus (c) Cationic form only
(b) Ground state (d) None of these
(c) First excited state 20. Which of the following has the maximum number of
(d) Infinite distance from the nucleus unpaired electron (atomic number of Fe 26)?
12. Which electronic level would allow the hydrogen atom (a) Fe (b) Fe (II) (c) Fe (III) (d) Fe (IV)
to absorb a photon but not to emit a photon? 21. Which quantum number is not related with Schroding-
(a) 3s (b) 2p (c) 2s (d) 1s er equation?
13. The third line in Balmer series corresponds to an (a) Principal (b) Azimuthal
electronic transition between which Bohr’s orbits in (c) Magnetic (d) Spin
hydrogen? 22. If l0 is the threshold wavelength for photoelectric
(a) 5 Æ 3 (b) 5 Æ 2 (c) 4 Æ 3 (d) 4 Æ 2 emission, l wavelength of light falling on the surface
14. Correct set of four quantum numbers for valence elec- of metal, and m mass of electron, then de Broglie wave-
tron of rubidium (Z = 37) is length of emitted electron is

IIT JEE PC-V1_02.indd 12 3/21/2017 5:15:11 PM


Join our telegram channel: @Jee_neet_topper

Atomic Structure 2.13

1 1 influence of electric field (E), magnetic field (B)


È h(ll0 ) ˘ 2 È h ( l0 – l ) ˘ 2 and V is potential difference applied across elec-
(a) Í ˙ (b) Í ˙
Î 2mc(l0 – l ) ˚ Î 2mcll0 ) ˚ trodes.
1 1
31. The quantum numbers of four electrons (e1 to e4) are
È h ( l – l0 ) ˘ 2 hll0 ˘ 2 given below.
(c) Í ˙ (d) ÈÍ ˙
Î 2 mc ll )
0 ˚ Î 2mc ˚ n l m s
23. It is known that atoms contain protons, neutrons and e1 3 0 0 +1/2
electrons. If the mass of neutron is assumed to half of e2 4 0 1 1/2
its original value whereas that of proton is assumed to e3 3 2 2 –1/2
be twice of its original value then the atomic mass of e4 3 1 –1 1/2
14
6C will be
The correct order of decreasing energy of these elec-
(a) Same (b) 25 % more trons is
(c) 14.28 % more (d) 28.5 % less (a) e4 > e3 > e2 > e1 (b) e2 > e3 > e4 > e1
24. The shortest wavelength of He+ in Balmer series is x. (c) e3 > e2 > e4 > e1 (d) none
Then longest wavelength in the Paschene series of Li+2 32. If radius of second stationary orbit (in Bohr’s atom) is
is R. Then radius of third orbit will be
36 x 16 x 9x 5x (a) R/3 (b) 9R (c) R/9 (d) 2.25 R
(a) (b) (c) (d)
5 7 5 9 33. The wavelength associated with a gold weighing 200 g
25. An electron in a hydrogen atom in its ground state ab- and moving at a speed of 5 m/h is of the order
sorbs energy equal to ionisation energy of Li+2. The (a) 10–10 m (b) 10–20 m
wavelength of the emitted electron is (c) 10 m–30
(d) 10–40 m
(a) 3.32 ¥ 10–10 m (b) 1.17 Å 34. If the nitrogen atom had electronic configuration 1s7,
(c) 2.32 ¥ 10–9 nm (d) 3.33 pm it would have energy lower than that of normal ground
26. In compound FeCl2, the orbital angular momentum state configuration 1s2 2s2 2p3, because the electrons
of last electron in its cation and magnetic moment (in would be closer to the nucleus. Yet 1 s7 is not observed
Bohr Magneton) of this compound are
because it violates
(a) ( 6), 35 (b) ( 6), 24 (a) Heisenberg uncertainty principle
(c) 0 (d) None of these (b) Hunds rule
27. An electron, a proton and an alpha particle have kinetic (c) Pauli’s exclusion principle
energy of 16 E, 4E and E respectively. What is the qual- (d) Bohr postulate of stationary orbits
itative order of their de Broglie wavelengths? 35. The total number of electrons in ‘Cu’ atom having
(a) le > lp = la (b) lp = la > le
magnetic quantum number (m = 0) will be
(c) lp > le > la (d) la < le >> lp
(a) 13 (b) 8 (c) 10 (d) 12
28. Given DH for the process Li (g) h Li+3 (g) + 3e– is
19800 kJ/mole and IE1 for Li is 520 then IE2 and IE1 of 36. Find the number of electrons with the value of azi-
Li+ are respectively (approx value) muthal quantum number ‘l’ = 1 for Cd2+.
(a) 11775, 7505 (b) 19280, 520 (a) 18 (b) 20 (c) 22 (d) 24
(c) 11775, 19280 (d) Data insufficient 37. Two bulbs A and B are emitting monochromatic light of
29. The ratio of difference in wavelengths of first and sec- wavelength such that A can just ionise H atoms and B
ond lines of Lyman series in H-like atom to difference in can just ionise He+ ions. If the power of A and B are 30
wavelength for second and third lines of same series is W and 40 W respectively, calculate the ratio of number
(a) 2.5 : 1 (b) 3.5 : 1 (c) 4.5 : 1 (d) 5.5 : 1 of photons emitted per second by bulb A to bulb B.
30. Which of the following statements is INCORRECT? (a) 4 (b) 5 (c) 3 (d) 7
e 38. In a sample of hydrogen atoms, electrons jump from
(a) ratio for canal rays is maximum for hydrogen
m 10th excited state to ground state. If x is the number of
ion. different ultraviolet radiations, y is the number of dif-
e ferent visible radiations and z is the number of different
(b) ratio for cathode rays is independent of the gas
m infrared radiations. The value of z – (x + y) is
taken. [Assume all the Balmer lines lie within visible region.]
(c) The nature of canal rays is dependent on the elec- (a) 17 (b) 18 (c) 19 (d) 36
trode material. 39. A 4pr2 y2 (r) vs r plotted for a H-orbital curve contains
e E2 ‘3’ maxima. If orbital contains 2 angular node then or-
(d) The ratio for electron is expressed as ,
m 2 B 2V bital is
when the cathode rays go undeflected under the (a) 5d (b) 4d (c) 5p (d) 4p

IIT JEE PC-V1_02.indd 13 3/21/2017 5:15:13 PM


Join our telegram channel: @Jee_neet_topper

2.14 Physical Chemistry-I for JEE (Main & Advanced)

40. In a hydrogen atom, in transition of electron, a photon (a) Angular momentum is an integral multiple of
of energy 2.55 eV is emitted. Then the change in wave- h
.
length of the electron is 2p
(a) 3.32 Å (b) 6.64 Å (b) The electron stationed in the orbit is stable.
(c) 9.97 Å (d) None of these (c) The path of an electron is circular.
41. The ratio of the wavelength of a proton and a-particle (d) The change in the energy levels of electron is con-
will be 1 : 2 if their tinuous.
(a) velocity of proton to velocity of a particle is in the 48. In which orbit of the hydrogen atom is the speed of the
ratio 1 : 8. electron maximum?
(b) velocity of proton to velocity of a particle is in the (a) n = 2 (b) n = 1 (c) n = 3 (d) n = 4
ratio 8 : 1. 49. For an electron, the product vn (velocity ¥ principal
(c) kinetic energy of proton to kinetic energy of a quantum number) will be independent of the
particle is in the ratio 64 : 1. (a) principal quantum number
(d) kinetic energy of proton to kinetic energy of a (b) velocity of the electron
particle is in the ratio 6 : 1. (c) energy of the electron
42. Choose the correct statement. (d) frequency of its revolution
(I) dyz orbital lies in the xz plane. 50. Assume that the potential energy of a hydrogen atom
(II) pz orbital lies along x-axis. in its ground state is zero. Then its energy in the first
(III) Lobes of Px orbital are at 90° with z-axis excited state will be
(IV) Lobes of dxy orbital are at 90° with z-axis. (a) 13.6 eV (b) 27.2 eV
(a) III, IV (b) II, III (c) 23.8 eV (d) 10.2 eV
(c) I, II (d) All of these 51. In an atom, two electrons move around the nucleus in
43. Choose the correct option regarding energy of empty circular orbits of radii R and 4R. The ratio of the time
orbitals. taken by them to complete one revolution is
n l m s (a) 1 : 4 (b) 4 : 1 (c) 1 : 8 (d) 8 : 7
1 52. The wavelength of a spectral line for an electronic tran-
(I) 4 0 0 +
2 sition is inversely related to
1 (a) the number of electrons undergoing the transition.
(II) 3 2 0 - (b) the nuclear charge of the atom.
2
(c) the difference in the energy of the energy levels
1
(III) 3 1 1 + involved in the transition.
2 (d) the velocity of electron undergoing the transition.
1 53. When the electron of a hydrogen atom jumps from
(IV) 3 0 0 -
2 n = 4 to n = 1 state, the number of spectral lines emit-
(a) I > IV (b) II > I ted is
(c) II > III (d) I = III (a) 15 (b) 6 (c) 3 (d) 4
(a) a, b, c (b) b, c 54. When the frequency of light incident on a metallic
(c) b, c, d (d) All of these plate is doubled, the K.E. of the emitted photoelectron
44. Find the product of multiplication of number of elec- will be
trons corresponding to (n + l) = 3 and l = 2 in Cu (29). (a) doubled
(a) 80 (b) 90 (c) 70 (d) 60 (b) halved
45. Which of the following may represent the possible (c) increased but more than doubled of the previous
quantum numbers for the last electron of Ga? K.E.
1 1 (d) Quantum number remains unchanged.
(I) 3, 1, +1, + (II) 4, 0, +1, + 55. The electrons, identified by quantum numbers n and l,
2 2
(i) n = 4, l = 1, (ii) n = 4, l = 0, (iii) n = 3, l = 2, and (iv)
1 1 n = 3, l = 1 can be placed in order of increasing energy,
(III) 4, 1, 0, – (IV) 4, 1, +1, +
2 2 from the lowest to highest, as
(a) III, IV (b) II, III (a) (iv) < (ii) < (iii) < (i) (b) (ii) < (iv) < (i) < (iii)
(c) III, IV (d) All of these (c) (i) < (iii) < (ii) < (iv) (d) (iii) < (i) < (iv) < (ii)
46. For which of the following species, Bohr’s theory is 56. The correct set of four quantum number for the outer-
not applicable? most electrons of rubidium (z = 37) is
(a) Be+3 (b) Li+2 (c) He+2 (d) H 1 1
47. Which of the following postulates does not belong to (a) 5, 0, 0, + (b) 5, 1, 0, +
2 2
Bohr’s model of the atom?

IIT JEE PC-V1_02.indd 14 3/21/2017 5:15:14 PM


Join our telegram channel: @Jee_neet_topper

Atomic Structure 2.15

1 1 69. Of the following transitions in hydrogen atom, the one


(c) 5, 1, 1, + (d) 6, 0, 0, + which gives an absorption line of maximum wave-
2 2
length is
57. The number of electrons in sulphur atom having n + l
(a) n = 1 to n = 2 (b) n = 3 to n = 8
= 3 is
(c) n = 2 to n = 1 (d) n = 8 to n = 3
(a) 2 (b) 4 (c) 6 (d) 8
70. Electromagnetic radiation with maximum wavelength
58. The orbital angular momentum of an electron in 2s-
is
orbital is
(a) ultraviolet (b) radiowave
(a) h/4p (b) zero
(c) X-ray (d) infrared
(c) h/2p (d) 2h /2p 71. The wave number of the first line in the Balmer se-
59. Magnetic moments of V(Z = 23), Cr(Z = 24), Mn(Z = ries of hydrogen atom is 15200 cm–1. What is the wave
25) are x, y, z. Hence number of the first line in the Balmer series of Be3+?
(a) x = y = z (b) x < y < z (a) 2.432 ¥ 105 cm–1 (b) 15200 cm–1
–1
(c) x < z < y (d) z y < x (c) 415200 cm (d) 215200 cm–1
60. The radial distribution curve of the orbital with double 72. The wavelength associated with a golf ball weighing
dumbbell shape in the fourth principle shell consists of 200 g and moving at a speed of 5 m/h is of which order
‘n’ nodes, n is (a) 10–10 m (b) 10–20 m
–30
(a) 2 (b) 0 (c) 1 (d) 3 (c) 10 m (d) 10–40 m
61. The increasing order (lowest first) for the values of e/m 73. If the following matter waves travel with equal veloc-
(charge/mass) for electron (e), proton (p), neutron (n) ity, the longest wavelength is that of
and alpha particle (a) is (a) electron (b) proton
(a) e, p, n, a (b) n, p, e, a (c) neutron (d) alpha particle
(c) n, p, a, e (d) n, a, p, e 74. If the velocity of hydrogen molecule is 5 ¥ 104 cm sec–1,
62. The ratio of charge and mass would be greater for then its de Broglie wavelength is
(a) Proton (b) Electron (a) 2 Å (b) 4 Å (c) 8 Å (d) 100 Å
(c) Neutron (d) Alpha 75. Correct set of four quantum numbers for the valence
63. Calcium (Ca) has atomic number 20 and mass number (outermost) electron of Ca
40. Which of the following statements is not correct (a) 4, 0, 0, + 1/2 (b) 5, 1, 0, + 1/2
about Ca atom? (c) 4, 1, 1, + 1/2 (d) 6, 0, 0, + 1/2
(a) The number of protons is same as the number of 76. The quantum numbers of most energetic electron in Ar
neutrons. atom when it is in first excited state is
(b) The number of electrons is same as the number of (a) 2, 1, 0, 1/2 (b) 4, 1, 1, ½
neutrons. (c) 4, 0, 0, 1/2 (d) 4, 1, 0, 1/2
(c) The number of protons is half of the number of 77. The outermost electronic configuration of the most
neutrons. electronegative element is
(d) The number of nucleons is double the number of (a) ns2np3 (b) ns2np4 (c) ns2np5 (d) ns2np6
neutrons. 78. Which of the following orbitals is associated with the
64. The triad of nuclei that is isotonic is angular nodes?
(a) s-orbials (b) p-orbitals
(a) 14 15 17
6 C, 7 N, 9 F (b) 12 14 19
6 C, 7 N, 9 F
(c) d-orbitals (d) f-orbitals
(c) 14 14 17
6 C, 7 N, 9 F (d) 14 14 19
6 C, 7 N, 9 F 79. The energy of the second Bohr orbit in the hydrogen
65. Rutherford’s experiment on scattering of particles atom is –3.41 eV. The energy of the second Bohr orbit
showed for the first time that the atom has of He+ ion would be
(a) electrons (b) protons (a) –0.85 eV (b) –13.64 eV
(c) nucleus (d) neutrons (c) –1.70 eV (d) –6.82 eV
66. The value of Bohr radius of hydrogen atom is 80. The ratio of energy of the electron in ground state of
(a) 0.529 ¥ 10–8 cm (b) 0.529 ¥ 10–10 cm hydrogen to the electron in first excited state of Be+3 is
–12
(c) 0.529 ¥ 10 cm (d) 0.529 ¥ 10–6 cm (a) 4 : 1 (b) 1 : 4 (c) 1 : 8 (d) 8 : 1
67. Ratio of radii of second and first Bohr orbits of H-atom 81. The ratio of the difference in energy of electron be-
is tween the first and second Bohr’s orbit to that between
(a) 2 (b) 4 (c) 3 (d) 5 second and third Bohr’s orbit is
68. The radius of which of the following orbit is same as 1 27 9 4
(a) (b) (c) (d)
that of the first Bohr’s orbit of hydrogen atom 3 5 4 9
(a) He+ (n = 2) (b) Li+2 (n = 2) 82. The first emission line in the atomic spectrum of hy-
2+
(c) Li (n = 3) (d) Be3+ (n = 2) drogen in the Balmer series appears at

IIT JEE PC-V1_02.indd 15 3/21/2017 5:15:15 PM


Join our telegram channel: @Jee_neet_topper

2.16 Physical Chemistry-I for JEE (Main & Advanced)

9 RH 7 RH 95. If uncertainty in position of an electron was zero, the


(a) cm –1 (b) cm –1 uncertainty in its momentum will be
400 144
h h
3RH 5 RH (a) zero (b) (c) (d) Infinity
(c) cm –1 (d) cm –1 2p 4p
4 36
83. What element has a H-like spectrum and whose lines 96. Which of the following represents the correct set of the
have wavelengths four times shorter than those of four quantum numbers of 4d-electrons?
atomic hydrogen? 1
(a) 4, 3, 2, + (b) 4, 2, 1, 0
(a) He (b) He+ (c) H (d) Li+2 2
84. The second line of Lyman series of H coincides with 1 1
(c) 4, 3, –2, (d) 4, 2, 1, -
the sixth line of Paschen series of an ionic species X. 2 2
Find X assuming R to be same for both H and X? 97. The first emission line in the atomic spectrum of hy-
(a) He+ (b) Li+2 (c) Li+ (d) H drogen in the Balmer Series appears at
85. The work function for a metal is 4 eV. To emit a photo 9 RH 7 RH
electron of zero velocity from the surface of the metal, (a) cm –1 (b) cm –1
400 144
the wavelength of incident light will be
(a) 2700 Å (b) 1700 Å 3RH 5 RH
(c) cm –1 (d) cm –1
(c) 5900 Å (d) 3100 Å 4 36
86. If l1 and l2 denote the de Broglie wavelength of two 98. The fourth line of the Balmer series corresponds to
particles with same masses but charges in the ratio of electron transition between which energy levels
1 : 2 after they are accelerated from rest through the (a) 6 and 2 (b) 5 and 2
same potential difference, then (c) 4 and 1 (d) 5 and 1
(a) l1 = l2 (b) l1 < l2 99. The ionisation potential of hydrogen atom is 13.6 eV.
(c) l1 > l2 (d) None of these The ionisation potential of Be3+ ion is
87. The velocity of electron of H-atom in its ground state is (a) 217.6 eV (b) 109.8 eV
2.2 ¥ 106 m/s. The de Broglie wavelength of this elec- (c) 54.4 eV (d) 136.0 eV
tron will be 100. If 10–17 J of light energy is needed by the interior of
(a) 0.33 nm (b) 23.3 nm human eye to see an object, the photons of green light
(c) 45.6 nm (d) 100 nm (l = 550 nm) needed to see the object are
88. The wavelength associated with a golf ball weighing (a) 27 (b) 28 (c) 29 (d) 30
0.200 g and moving at a speed of 5 m/h is of the order 101. The energy of a photon having wavelength 700 nm is
(a) 10–10 m (b) 10–27 m (a) 1.77 eV (b) 2.47 eV
–30 (c) 700 eV (d) 3.57 eV
(c) 10 m (d) 10–40 m
102. A 1-kW radio transmitter operates at a frequency of
89. An electron in a H-like atom is in an excited state. It
880 Hz. How many photons per second does it emit?
has a total energy of –3.4 eV. Calculate the de Broglie’s
(a) 1.71 ¥ 1021 (b) 1.71 ¥ 1030
wavelength? 23
(c) 6.02 ¥ 10 (d) 2.85 ¥ 1026
(a) 66.5 Å (b) 6.66 Å
103. The ratio of the e/m values of a proton and an a-particle
(c) 60.6 Å (d) 6.06 Å
is
90. Non-directional orbital is
(a) 2 : 1 (b) 1 : 1 (c) 1 : 2 (d) 1 : 4
(a) 3s (b) 4f (c) 4d (d) 4p
104. Which particle among the following will have the
91. The ionisation energy of a hydrogen atom in terms of
smallest de Broglie wavelength, assuming that they
Rydberg constant (RH) is given by the expression have the same velocity?
(a) RHhc (b) RHc (a) A positron (b) A photon
(c) 2RHhc (d) RHhcNA (c) An a-particle (d) A neutron
92. The ratio of the energies of photons of 2000 Å to that of 105. The energy needed to excite a hydrogen atom from its
4000 Å is ground state to its third excited state is
1 1 (a) 12.1 eV (b) 10.2 eV
(a) 2 (b) 4 (c) (d)
2 4 (c) 0.85 eV (d) 12.75 eV
93. For a ‘d’ electron, the orbital angular momentum is 106. The wavelength of the third line of the Balmer series
(a) 6 (b) 2 (c)  (d) 2 for a hydrogen atom is
94. The energy of an electron in the first Bohr orbit of H- 21 100
(a) (b)
atom is –13.6 eV. The possible energy value of the ex- 100R• 21R •
cited state in the Bohr orbit of H-atom is 21R• 100 R•
(a) –4.2 eV (b) –3.4 eV (c) (d)
100 21
(c) –6.8 eV (d) +6.8 eV

IIT JEE PC-V1_02.indd 16 3/21/2017 5:15:16 PM


Join our telegram channel: @Jee_neet_topper

Atomic Structure 2.17

107. When the value of the azimuthal quantum number is 119. The energy for an electron is an orbit of hydrogen atom
3, the maximum and the minimum values of the spin 13.6
multiplicities are is given by, En = - 2 Z 2 eV . Calculate the energy of
n
(a) 4, 3 (b) 8, 1 (c) 1, 3 (d) 8, 2 the orbit having a radius 9r1 where r1 is the radius of
108. The energy of an electron in the first Bohr orbit of H- the first orbit?
atom is –13.6 eV. The possible energy value of the 3rd (a) –1.51 eV (b) 10.2 eV
excited state for electron in Bohr orbit of hydrogen is (c) 13.6 eV (d) 1.36 eV
(a) –3.4 eV (b) – 0.85 eV 120. An electron in a Bohr orbit of hydrogen atom with the
(c) – 6.8 eV (d) + 6.8 eV quantum number n2 has an angular momentum 4.2176
109. For the hydrogen atom, the energy of the electron is ¥ 10–34 kg m2 s–1. If electron drops from this level to
defined by the factor En = –13.58/n2 eV. Here n is posi- the next lower level, find the wavelength of this line.
tive integer. The minimum quantity of energy which it
(a) 18.75 ¥ 10–7 m (b) 1.87 ¥ 10–7 m
can absorb in its primitive stage is –7
(c) 187.5 ¥ 10 m (d) 0.187 ¥ 10–7 m
(a) 1.00 eV (b) 3.39 eV
121. The maximum wavelength of light that can excite an
(c) 6.79 eV (d) 10.19 eV
110. The radius of Bohr’s first orbit in hydrogen atom is electron from first to third orbit of hydrogen atom is
0.053 nm. The radius of second orbit in He+ can be (a) 487 nm (b) 170 nm
(a) 0.0265 nm (b) 0.0530 nm (c) 103 nm (d) 17 nm
(c) 0.106 nm (d) 0.2120 nm 122. The work function for a metal is 4 eV. To emit a photo
111. The ratio of the radii of the first three Bohr orbits is electron of zero velocity from the surface of the metal,
(a) 1 : 0.5 : 0.33 (b) 1 : 2 : 3 the wavelength of incident light should be (nearly)
(c) 1 : 4 : 9 (d) 1 : 8 : 27 (a) 2700 Å (b) 1700 Å
112. An electron has wavelength 1 Å. The potential by (c) 5900 Å (d) 3100 Å
which the electron is accelerated will be 123. The total energy of the electron in the hydrogen atom in
(a) 0.0926 V (b) 0.0502 V the ground state is –13.6 eV. The K.E. of this electron is
(c) 0.0826 V (d) 150 V (a) 13.6 eV (b) Zero
113. Atomic number of chromium is 24, then Cr3+ will be (c) –13.6 eV (d) 6.8 eV
(a) diamagnetic (b) paramagnetic 124. Which of the following sets of quantum numbers repre-
(c) ferromagnetic (d) None of these sents the highest energy of an atom?
114. Which of the following arrangement of two electrons 1
in two degenerated orbitals is not possible at all? (a) n = 4, l = 0, m = 0, s = +
2
(a) (b) 1
(b) n = 3, l = 0, m = 0, s = +
2
(c) (d) All 1
(c) n = 3, l = 1, m = 1, s = +
2
115. A compound of vanadium possesses a magnetic mo- 1
ment of 1.73 BM. The oxidation state of vanadium in (d) n = 3, l = 2, m = 1, s = +
2
this compounds is
125. Which option gives the values of the quantum numbers
(a) 1 (b) 2
for the 21st electron of scandium (Z = 21)?
(c) 4 (d) Cannot be predicted
116. Uncertainty in position and momentum are equal. Un- 1 1
(a) 3, 1, 1, + (b) 3, 2, 2, +
certainty in velocity is 2 2
(a) h / p (b) h /2p 1 1
(c) 3, 2, –3, – (d) 3, 1, 2, –
(c) 1/2m h / p (d) None of these 2 2
117. The second line of Lyman series of H coincides with 126. The number of unpaired electrons in Mn4+ (Z = 25) is
the sixth line of Paschen series of an ionic species X. (a) four (b) two (c) five (d) three
Find X assuming Rydberg constant to be same for both 127. The value of the magnetic moment of a particular ion is
H and X? 2.83 Bohr magneton. The ion is
(a) Li+2-ion (b) Li+-ion (a) Fe2+ (b) Ni2+ (c) Mn2+ (d) Co3+
+
(c) He -ion (d) None of these 128. The number of spherical nodes in 3p orbitals is
118. Two particles A and B are in motion. If the wavelengths
(a) one (b) three (c) two (d) zero
associated with particle A is 5 ¥ 10–8 m, calculate the
129. Which of the following electronic configurations have
wavelength of particle B if momentum of B is half of A.
zero spin multiplicity?
(a) 10–8 m (b) 10–7 m (c) 10–5 m (d) 10–3m

IIT JEE PC-V1_02.indd 17 3/21/2017 5:15:18 PM


Join our telegram channel: @Jee_neet_topper

2.18 Physical Chemistry-I for JEE (Main & Advanced)

140. From the following observations, predict the type of


(a) (b) orbital.
Observation 1: x y plane acts as nodal plane.
Observation 2: The angular function of the orbital in-
(c) (d) none of these
tersect the three axis at origin only.
130. The fraction of volume occupied by the nucleus with Observation 3: R2(r)v/s r curve obtained for the orbital
respect to the total volume of an atom is is
(a) 10–15 (b) 10–5 (c) 10–30 (d) 10–10 R2(r)
131. Which of the following is iso-electronic with neon-
(a) O2– (b) F+ (c) Mg (d) Na
132. The MRI (magnetic resonance imaging) body scanners
used in hospitals operate with 400 MHz radio frequen-
cy energy. The wavelength corresponding to this radio
r
frequency is
(a) 5pz (b) 6dxy (c) 6dx2 – y2 (d) 6 dyz
(a) 0.75 m (b) 0.75 cm (c) 1.5 m (d) 2 cm
133. Radius of Nucleus of atom having Atomic mass = 64 141. Consider the following nuclear reactions involving X
will be and Y:
(a) 5 fm (b) 9 fm X h Y + 42 He
(c) 10 fm (d) 85 fm
Y h 8O18 + 1H1
134. What is likely to be principal quantum number for a
If both neutrons as well as protons in both sides are
circular orbit of diameter 20 nm of the hydrogen atom
conserved in nuclear reaction then identify period num-
if we assume Bohr orbit be the same as that represented
ber of X and moles of neutrons in 4.6 g of X.
by the principal quantum number?
(a) 3, 2.4 NA (b) 3, 2.4
(a) 10 (b) 14 (c) 12 (d) 16
(c) 2, 4.6 (d) 3, 0.2 NA
135. Choose the correct relationship.
142. Electromagnetic radiations having l = 310 Å are sub-
(a) E1 of H = 1/2 E2 of He+ = 1/3 E3 of Li2+ = 1/4 E4
jected to a metal sheet having work function = 12.8
of Be3+
eV. What will be the velocity of photoelectrons with
(b) E1(H) = E2(He+) = E3(Li2+) = E4(Be3+)
maximum kinetic energy.
(c) E1(H) = 2E2(He+) = 3E3(Li2+) = 4E4(Be3+)
(a) 0, no emission will occur
(d) No relation
(b) 2.18 ¥ 106 m/s
136. If the value of E = – 78.4 kcal/mole, the order of the
orbit in hydrogen atom is (c) 2.18 2 ¥ 106 m/s
(a) 4 (b) 3 (c) 2 (d) 1 (d) 8.72 ¥ 106 m/s
137. If velocity of an electron in first orbit of H atom is V, 143. If in Bohr’s model, for unielectronic atom, time period
what will be the velocity of 3rd orbit of Li+2? of revolution is represented as Tn, z where n represents
(a) V (b) V/3 (c) 3 V (d) 9 V shell number and z represents atomic number then the
138. In a certain electronic transition in the hydrogen atoms value of T1, 2 : T2, 1 will be
from an initial state (A) to a final state (B), the differ- (a) 8 : 1 (b) 1 : 8
ence in the orbit radius |r1 – r2| is 624 times the first (c) 1 : 1 (d) None of these
Bohr radius. Identify the transition. 144. Which of the following orbital is non-directional?
(a) 5 Æ 1 (b) 25 Æ 1 (c) 8 Æ 3 (d) 7 Æ 5 (a) s (b) p
139. Match the following: (c) d (d) All of these
(a) Energy of ground state of (i) –6.04 eV 145. Uncertainty in position is twice the uncertainty in mo-
He+ mentum Uncertainty in velocity is
(b) Potential energy of I orbit (ii) –27.2 eV h 1 h
(a) (b)
of H-atom p 2m p
(c) Kinetic energy of II excit- (iii) 8.7 ¥ 10–18 J 1 h
ed state of He+ (c)  (d)
2m 4p
(d) Ionisation potential of He+ (iv) –54.4 eV 146. For which orbital angular probability distribution is
(a) A – (i), B – (ii), C – (iii), D – (iv) maximum at an angle of 45° to the axial direction?
(b) A – (iv), B – (iii), C – (ii), D – (i) (a) dx2 – y2 (b) dz2
(c) A – (iv), B – (ii), C – (i), D – (iii) (c) dxy (d) Px
(d) A – (ii), B – (iii), C – (i), D – (iv) 147. Which orbit will be the first to have ‘g’ subshell?
(a) 3rd (b) 4th (c) 5th (d) 6th

IIT JEE PC-V1_02.indd 18 3/21/2017 5:15:19 PM


Join our telegram channel: @Jee_neet_topper

Atomic Structure 2.19

148. Frequency of revolution of e– in shell (II) Ti3+, Cr+, Sc2+ ions are diamagnetic.
4 3 (III) Value of (n + l + m) for last electron of Mg is 3.
pr (IV) The value of Zeff for 3s electron of Cl is 10.9.
2p r
(a) (b) 3 (a) I, IV (b) I, III
V V
(c) III, IV (d) All of these
V qV 6. An element has mass number of 23 and its unipositive
(c) (d)
2p r 2p r ion has electronic configuration 1s2 2s2 2p6. Select the
149. The decreasing order of energy of the 3d, 4s, 3p, 3s correct statement(s).
orbitals is (I) It is isotonic with Mg24.
(a) 3d > 3s > 4s > 3p (b) 3s > 4s > 3p > 3d (II) The ratio of n to l for last electron in atom of above
(c) 3d > 4s > 3p > 3s (d) 4s > 3d > 3s > 3p element is 2.
150. If n and l are respectively the principle and azimuthal (III) Atom of the above element is isoelectronic with
quantum numbers, then the expression for calculating H2O.
the total number of electrons in any orbit is (IV) Atom of the above element is paramagnetic.
l=n l = n -1 (a) I, IV (b) II, III (c) III, IV (d) All
(a) Â 2(2l + 1) (b) Â 2(2l + 1) 7. If the Aufbau principle is not followed and subshells
l =1 l =1 of a shell are progressively filled according to their en-
l = n +1 l = n -1
ergy, which of the following elements is paramagnetic?
(a) Mg (b) Ca (c) Ni (d) Zn
(c) Â 2(2l + 1) (d) Â 2(2l + 1)
8. Find the total number of species having magnetic mo-
l =0 l =0
ment value of 2.84 B.M. from following species:
Fe+2, Cr, Cr3+, Ti2+, Mn2+, V3+
LEVEL II (a) 2 (b) 3 (c) 4 (d) 5
9. Select the correct statement(s).
1. Select the correct statement for Ne. 1
(I) Maximum number of electrons having s = + in
(a) It is not isoelectronic with H2O. 2
sulphur is 9.
(b) Last electron enters in s-orbital.
(II) If electron has zero magnetic quantum number
(c) The value of ‘m’ must be zero for last electron.
then it must be present in s-orbital.
(d) The value of ‘l’ must be ‘1’ for last electron.
(III) All isoelectronic ions have same value of Zeff.
2. Give the correct order of initials True (T) or False (F)
(IV) Number of subshell present in a nth shell is equal to
for the following statements:
‘n’.
(I) Number of electrons having ‘l = 0’ is 10 in Pd.
(a) I, IV (b) II, III
(II) The value of Zeff for 3d electron of Cr and 3d elec-
(c) III, IV (d) All of these
tron of Mn is same as the number of electron in ‘d’
10. Total number of electrons having l = 2 in Fe+2 accord-
subshell of Cr and Mn are same.
ing to Aufbau principle (n + l rule) is
(III) Multiplicity of Fe is equal to Ni+2.
(a) 5 (b) 4 (c) 3 (d) 6
Ê lˆ 11. Similar to quantum number n, l, m a set of new quan-
(IV) Value of ÁË ˜¯ for last electron of element having
n tum numbers was introduced with similar logic but dif-
atomic number 57 is 0.4. ferent values defined as
(a) T T T T (b) F T T T x = 1, 2, 3... all (+ve) integral values
(c) T F T F (d) F F F T y = (x – 1) (x – 2)(x – 3)... No (–ve) value
3. ‘A+2’ is isoelectronic with ‘CO2’ and has (Z+2) neu- z = +(y + 1/2) to – (y + 1/2) in integral steps
tron (Z is atomic number of A), then select the correct For each value of (z), there will be two electrons, if (x + y)
option(s). rule is followed similar to (n + l) rule then number of
(I) Mass number of A+2 is 50. electron in third shell for Zn (Atomic number = 30) is
(II) Last electron enters in ‘s’ orbital in element ‘A’. (a) 12 (b) 13 (c) 14 (d) 15
(III) Number of unpaired electron in ‘A’ is 4. 12. Select the incorrect statement(s) (According to Bohr
(IV) During formation of A+2, electron(s) are removed model).
from ‘s’ as well as ‘d’ subshell. -2p 2 K 2 Z 2e 4 m
(a) I, IV (b) II, III (c) III, IV (d) All (a) Total energy =
n2h2
4. Calculate ‘Q’ for last electron of Ga
where Q = n + l + maximum possible value of ‘m’. n2h2
(b) Radius of orbit =
(a) 6 (b) 8 (c) 7 (d) 5 4p 2 KZe 2 m
5. Which of the following is / are true? 2p KZe 2
(I) Multiplicity in Fe3+ is greater than that in Co3+. (c) Velocity of electron in an orbit =
nh

IIT JEE PC-V1_02.indd 19 3/21/2017 5:15:20 PM


Join our telegram channel: @Jee_neet_topper

2.20 Physical Chemistry-I for JEE (Main & Advanced)

(d) Frequency of revolution of an electron in an orbit to stop photocurrent completely is 0.24 V. The kinetic
n3 h3 energy of ejected electron is
= (I) 0.24 eV (II) 0.20 eV
4p K 2 Z 2e 4 m
(III) 0.10 eV (IV) 2.4 eV
13. The electrons in a sample of gaseous Li2+ ions which
(a) I, II, III (b) II, III, IV
are initially present in a particular excited state makes
(c) I, III, IV (d) All of these
a transition to a lower level. The emitted photons are
19. The distance of spherical nodes from nucleus for the
absorbed by a sample of H-atoms which are present given orbital are
in their ground state. This sample of H-atoms on de- 3/2
excitation gives maximum 6 different types of spectral 1 ÊZˆ
Y radial = [(s 2 - 4s + 3)] exp(- s /2)
lines. Find the quantum number of initial excited state 9 2 ÁË a0 ˜¯
of Li2+ ions. 2Zr
where a0 and Z are the constants and s = .
(a) 11 (b) 12 (c) 13 (d) 14 a0
14. The radial wave function for 1s electron in H-atom is a0 1 a0
(a) Zero, infinity (b)
,
2 Z 2 Z
R = 3/2 e - r / a0 where a0 = radius of first orbit of H-at-
a0 3 a0 1 a0 a 3 a0
(c) , (d) 0 ,
om. The ratio of probability of 1s electron in hydrogen 2 Z 2 Z Z 2 Z
a 20. The difference in angular momentum associated with
atom at distance a0 from nucleus to that at distance 0 the electron in two successive orbits of hydrogen atoms
from nucleus is 2
is
1 h h h (n - 1)h
(a) equal (b) times (a) (b) (c) (d)
e p 2p 2 2p
4 e 21. The shortest l for the Lyman series of hydrogen atom
(c) times (d) times is … (Given: RH = 109678 cm–1)
e 4
(a) 911.7 Å (b) 700 Å
15. Which of the following has correct the matching of (c) 600 Å (d) 811 Å
curve and orbital? 22. The velocity of an electron in the first Bohr orbit of a
I II III
hydrogen atom is 2.19 ¥ 106 ms–1. Its velocity in the
4pr2Ry2

4pr2Ry2

4pr2Ry2

second orbit would be


(a) 1.10 ¥ 106 ms–1 (b) 4.38 ¥ 106 ms–1
5 –1
r r r (c) 5.5 ¥ 10 ms (d) 8.76 ¥ 106 ms–1
(I) I (2p) II (1s) III (4p) 23. The atomic spectrum of Li+2 – ion arises due to the
(II) I (3p) II (3d) III (3s) transition of an electron from n2 to n1. If n1 + n2 = 4 and
(III) I (4d) II (2p) III (5d) (n2 – n1) = 2, find the wavelength of the third line of
(IV) I (2s) II (4f) III (3d) this series in Li+2-ion.
(a) II, III (b) I, III (c) III, IV (d) All (a) 1.08 nm (b) 10.8 nm
16. If the position of an electron can be determined within (c) 108 nm (d) 1080 nm
±0.0001 Å precision when momentum of electron is 24. If the wavelength is equal to the distance travelled by
h the electron in one second, then
, where a0 is 0.529 Å, find the ratio of minimum h h
2p a0 (a) l = (b) l =
uncertainty in momentum to given momentum. p p
[h = 6.626 ¥ 10–34] h h
(a) 2423 (b) 2645 (c) 2768 (d) 2357 (c) l = (d) l =
p m
17. An unknown particle having double charge as proton
25. According to the Schrodinger model, nature of electron
moves with wavelength l. It is accelerated from the
in an atom is
V (a) Particles only
rest through a potential difference of volts, while
8 (b) Wave only
proton itself moves with same wavelength l when ac- (c) Both simultaneously
celerated from rest through a potential difference ‘V’ (d) Sometimes waves and sometimes particle
volts. The particle is 26. Which of the following describes orbital?
(a) He+ (b) Li2+ ion (c) He2+ (d) Be2+ (a) y (b) y2
18. When photons of wavelength l = 253.7 nm is subjected 2
(c) |y |y (d) All of these
on Cu plate (work function = 4.65 eV) then photoelec- 27. In order to have the same wavelength for the electron
trons are emitted. The magnitude of potential required (mass me) and the neutron (mass mn), their velocities

IIT JEE PC-V1_02.indd 20 3/21/2017 5:15:22 PM


Join our telegram channel: @Jee_neet_topper

Atomic Structure 2.21

should be in the ratio (electron velocity/neutron veloc- (a) l1 + l2 + l3 = 0 (b) l3 = l1 + l22


ity) ll
(a) mn/me (b) mn ¥ me (c) me/mn (d) one (c) l3 = l1 + l2 (d) l3 = 1 2
l1 + l2
28. Which of the following is true about y
39. The wavelengths of photons emitted by electron transi-
(a) y represents the probability of finding an electron
tion between two similar levels in H and He+ are l1 and
around the nucleus.
l2 respectively. Then
(b) y represent the amplitude of the electron wave.
(a) l2 = l1 (b) l2 = 2l1
(c) Both A and B
(c) l2 = l1/2 (d) l2 = l1/4
(d) None of these
40. If first ionisation potential of an atom is 16 V, then the
29. Consider an electron in the nth orbit of a hydrogen atom
first excitation potential will be
in the Bohr model. The circumference of the orbit can
(a) 10.2 V (b) 12 V (c) 14 V (d) 16 V
be expressed in terms of the de Broglie wavelength l of
41. In which transition minimum energy is emitted?
the electron as
(a) • Æ 1 (b) 2 Æ 1
(a) (0.529) nl (b) nl (c) 3 Æ 2 (d) n Æ (n – 1) (n ≥ 4)
(c) (13.6) l (d) nl 42. Number of visible lines when an electron returns from
30. A particle X moving with a certain velocity has a de fifth orbit to ground state in H spectrum is
Broglie wavelength of 1A°. If particle Y has a mass of (a) 5 (b) 4 (c) 3 (d) 10
25% that of X and velocity 75% that of X, de Broglie’s
wavelength of Y will be
(a) 3 A° (b) 5.33 A° (c) 6.88 A° (d) 48 A° LEVEL III
31. What are the values of the orbital angular momentum
of an electron in the orbitals 1s, 3s, 3d and 2p? 1.
(a) 0, 0, 6, 2 (b) 1, 1, 4, 2 Column-I Column-II
(c) 0, 1, 6, 3 (d) 0, 0, 20, 6 (A) (p) s-orbital
32. If m = magnetic quantum number and l = azimuthal
y2r

quantum number, then


(a) m = l + 2 (b) m = 2l2 + 1
r
m –1
(c) l = (d) l = 2m + 1
2
33. The number of unpaired electrons in Mn4+ (Z = 25) is (B) (q) p-orbital
(a) Four (b) Two (c) Five (d) Three
r
4pr2y 2

34. After np orbitals are filled, the next orbital filled will be
(a) (n + 1) s (b) (n + 2) p
(c) (n + 1) d (d) (n + 2) s
35. The value of the magnetic moment of a particular ion is r–
2.83 Bohr magneton. The ion is
(a) Fe2+ (b) Ti (c) Mn2+ (d) Co3+ (C) (r) d-orbital
36. In Bohr’s model of the hydrogen atom, the ratio be-
y2 r

tween the period of revolution of an electron in the or-


bit of n = 1 to the period of the revolution of the elec-
tron in the orbit n = 2 is
(a) 1 : 2 (b) 2 : 1 (c) 1 : 4 (d) 1 : 8 r
37. Let u1 be the frequency of the series limit of the Lyman
(D) (s) f-orbital
series, u2 be the frequency of the first line of the Lyman
y2 r

series, and u3 be the frequency of the series limit of the


Balmer series.
(a) u1 – u2 = u3 (b) u2 – u1 = u3
r
(c) u3 = 1/2 (u1 – u3) (d) u1 + u2 = u3
38. The energies of energy levels A, B and C for a given (t) Radial
atom are in the sequence EA < EB < EC. If the radia- probability
tions of wavelengths l1, l2 and l3 are emitted due to distribution
the atomic transitions C to B, B to A and C to A respec- function.
tively then which of the following relations is correct?

IIT JEE PC-V1_02.indd 21 3/21/2017 5:15:25 PM


Join our telegram channel: @Jee_neet_topper

2.22 Physical Chemistry-I for JEE (Main & Advanced)

(a) (I) P (II) P, Q, R, S, T (III) Q, R, S (IV) P 6.


(b) (I) Q (II) Q, R, S, T (III) P, Q, R, S (IV) P Column-I Column-II
(c) (I) P, R (II) P, Q, R, S (III) Q, R (IV) S (A) Aufbau principle (p) Line spectrum in
(d) (I) P, Q, R (II) Q, S, T (III) P, Q, S (IV) R visible region
2. Matrix matching
(B) de Broglie (q) Orientation of an
Column-I Column-II electron in an orbital
(C) Angular (r) Photon
(A) Lyman series (p) Visible region momentum
(B) Balmer series (q) Infrared region (D) Hund’s rule (s) l = h/mn
(E) Balmer series (t) Electronic
(C) Paschen series (r) Utraviolet region
configuration
(D) Brackett series (s) n2 = 4 to n2 = 3 (F) Planck’s law (u) mvr
(t) n2 = 5 to n2 = 1 7.
Column-I Column-II
3. Matrix matching
(A) Cathode rays (p) Helium nuclei
Column-I Column-II
(B) dumb bell (q) Uncertainty principle
(A) 2p orbital (p) Number of spherical nodes (C) Alpha particles (r) Electromagnetic radiation
=0
(D) Moseley (s) p-orbital
(B) 3d orbital (q) Number of nodal plane = 0
(E) Heisenberg (t) Atomic number
(C) 2s orbital (r) Orbital angular momentum (F) X-ray (u) Electrons
number = 0
8. Frequency = f, Time period = T, Energy of nth orbit =
(D) 4f orbital (s) Azimuthal quantum num-
En, radius of nth orbit = rn, Atomic number = Z, Orbit
ber = 0
number = n:
4. Matrix matching Column-I Column-II
Column-I Column-II (A) f (p) n3
(A) Radius of nth orbit (p) Inversely propor- (B) T (q) Z2
tional to z 1
(C) En (r)
(B) Energy of nth orbit (q) Integral multiple n2
h 1
of (D) (s) Z
2p rn
(C) Velocity of electron (r) Proportional to n2
9.
in the nth orbit
Column-I Column-II
(D) Angular momentum (s) Inversely propor-
of electron tional to n (A) Lyman series (p) Maximum number
of spectral line ob-
(t) Proportional to Z2 served = 6
5. Matrix matching (B) Balmer series (q) Maximum number
of spectral line ob-
Column-I Column-II served = 2
(A) H (p) Radius of 4th orbit 0.53 ¥ 4 Å (C) In a sample 5 Æ 2 (r) Second line has
(B) He+ (q) Energy of second orbit = –13.6 eV 8R
wave number
(C) Be3+ (r) Radius of second orbit = 0.53 ¥ > 9
4Å (D) In a single isolat- (s) Second line has
(D) Li2+ (s) Velocity of electron in the third ed H-atom for 3 3R
wave number
orbit Æ 1 transition 4
(t) Energy of fourth orbit = –13.6 eV (t) Total number of
spectral line is 10

IIT JEE PC-V1_02.indd 22 3/21/2017 5:15:26 PM


Join our telegram channel: @Jee_neet_topper

Atomic Structure 2.23

10. Column I and Column II contain data on Schrodinger 15. When electrons make transition to ground state, the
Wave-Mechanical model, where symbols have their largest wavelength (in nm) in the emitted radiations is
usual meanings. Match the columns. [Given : “E” represents energy level in eV]
Column-I Column-II 1240 1240
(a) (b)
(A) (p) 4s | E6 - E5 | | E2 - E1 |
yr

1240 1240
(c) (d)
r | E6 - E1 | | E5 - E4 |
(B) yr24pr2 (q) 5px Passage 3
The magnetic quantum number is denoted by letter m, and for
a given value of l, it can have all the values ranging from –l to
+l including zero of l, m has 2l + 1 values. For example, if l =
(C) Y(q, f ) = K (independent of (r) 3s 2, m can values, i.e., m = –2, –1, 0, +1, +2. This implies that
q and f ) there are five different orientations of the d-subshell. In other
words, d-subshell has five d-orbitals.
(D) atleast one angular node is (s) 6dxy
16. How many electrons can fit into the orbitals that com-
present
prise the third quantum shell n = 3?
(a) 2 (b) 8 (c) 18 (d) 32
Passage 1
17. The maximum number of electron that can be accom-
In an atom, when an electron jumps from higher energy level
modated in f-shell is
to lower energy level, it emits energy in the form of electro-
(a) 2 (b) 8 (c) 18 (d) 14
magnetic radiations. When these electromagnetic radiations
18. Which of the following is correct for 2p-orbitals?
are passed through a prism and received on a photographic
(a) n = 1, l = 2 (b) n = 1, l = 0
film, some lines are observed and those lines are called spec-
(c) n = 2, l = 0 (d) n = 2, l = 1
tral lines.
For hydrogen-like species when jump takes from any ex- Passage 4
cited state to ground state (n = 1), line produced in that case A German physicist gave a principle about the uncertainties
is called a Lyman series line. in simultaneous measurement of position and momentum of
If transition occurs from third or above levels to second small particles. According to that physicist, it is impossible to
level, then corresponding lines produced are called Balmer measure simultaneously the position and momentum of small
lines. particle simultaneously with absolute accuracy or certainty. If
Similarly for next level, it is called Paschen series line. an attempt is made to measure any one of these two quantities
11. In a gaseous sample 3 atoms of a Hydrogen-like spe- with higher accuracy, the other becomes less accurate. The
cies are present and all atoms are in fourth excited state product of the uncertainty in position (Dx) and uncertainty
then calculate the maximum number of different spec- momentum (Dp) is always constant and is equal to or greater
tral lines that can be produced. h
than , where h is Plank’s constant, i.e.,
(a) 10 (b) 7 (c) 12 (d) 8 4p
h
12. If there are 3 atoms of a Hydrogen like specie in second, ( Dx)( Dp ) ≥
third and fourth excited state respectively, then how 4p
many maximum total different Balmer and Paschen 19. Uncertainty in position is twice the uncertainty in mo-
lines can be produced? mentum. Uncertainty velocity is
(a) 2 (b) 3 (c) 4 (d) 5 h 1 h
Passage 2 (a) (b)
p 2m p
Alpha particles capture free moving electrons having wave-
length 0.25 nm and form excited H-like species. In this excit- 1 1 h
(c) h (d)
ed state, electron has circumference 6 times the wavelength 2m 2 2m p
of electron. Bohr’s quantisation rule is applicable. 20. The uncertainty in position of an electron (m = 9.1 ¥
13. The energy of photon emitted in this process will be 10–28 gm) moving with a velocity 3 ¥ 104 cm/s accurate
(a) 20.49 eV (b) 22.49 eV up to 0.001% will be
(c) 25.51 eV (d) 27.51 eV (a) 3.84 cm (b) 1.92 cm
14. What is the number of maximum possible spectral lines (c) 7.68 cm (d) 5.76 cm
obtained in Balmer series, when electron makes transi- 21. If uncertainty in the position of an electron is zero, the
tion to ground state? uncertainty in its momentum would be
(a) 15 (b) 4 (c) 6 (d) 10 (a) zero (b) < h/4p
(c) > h/4p (d) Infinite

IIT JEE PC-V1_02.indd 23 3/21/2017 5:15:28 PM


Join our telegram channel: @Jee_neet_topper

2.24 Physical Chemistry-I for JEE (Main & Advanced)

Passage 5 1 Ê 1 1ˆ
It is tempting to think that all possible transitions are permis- n= = RH ◊ z 2 Á 2 - 2 ˜ …(i)
l Ë n1 n2 ¯
sible, and that an atomic spectrum arises from the transition
of an electron from any initial orbital to any other orbital. 2p 2 m k 2e 4
where RH = = Rydberg constant for H-atom
However, this is not so, because a photon has an intrinsic h 3c
h where the terms have their usual meanings.
spin angular momentum of 2 corresponding to S =
2p Considering the nuclear motion, the most accurate would
1 although it has no charge and no rest mass. On the other have been to replace mass of electron (m) by the reduced
mass (m) in the above expression, defined as
hand, an electron has two types of angular momentum: or-
m¢ ◊ m
h m=
bital angular momentum, L = l (l + 1) and spin angular m¢ + m
2p
hˆ where mf = mass of nucleus.
Ê
momentum, Ls Á = s ( s + 1) ˜ arising from orbital mo- For Lyman series: n1 = 1 (fixed for all the lines) while n2
Ë 2p ¯
= 2, 3, 4, … infinite for successive lines, i.e., first, second,
tion and spin motion of electron, respecitvely. The change
third … infinite lines, respectively. For Balmer series: n1 = 2
in angular momentum of the electron during any electronic
(fixed for all the lines) while n2 = 3, 4, 5 … infinite for suc-
transition must compensate for the angular momentum car-
cessive lines.
ried away by the photon. To satisfy this condition, the differ-
25. The ratio of the wave numbers for the highest energy
ence between the azimuthal quantum numbers of the orbitals
transition of e– in Lyman and Balmer series of H-atom
within which transition takes place must differ by one. Thus,
is
an electron in a d-orbital (l = 2) cannot make a transition into
(a) 4 : 1 (b) 6 : 1 (c) 9 : 1 (d) 3 : 1
an s-orbital (l = 0) because the photon cannot carry away
26. If proton in H-nucleus be replaced by positron having
enough angular momentum. An electron as is well known,
the same mass as that of proton but same charge as that
possesses four quantum numbers n, l, m and s. Out of these
of proton, then considering the nuclear motion, the
four, l determines the magnitude of orbital angular momen-
wave number of the lowest energy transition of He+ ion
tum (mentioned above) while m determines its Z-component
in Lyman series will be equal to
Ê hˆ (a) 2 RH (b) 3 RH (c) 4 RH (d) RH
as m Á ˜ . The permissible values of only integers from –l
Ë 2p ¯
Passage 7
to +l. While those for l are also integers starting from 0 to (n
The sum of spins of all electrons is the total spin(S) and
– 1). The values of l denotes the subshell. For l = 0, 1, 2, 3,
2S + 1 and is called spin multiplicity of the electronic config-
4… the sub-shells are denoted by the symbols s, p, d, f, g….
uration. Hund’s rule defines the ground state configuration of
respectively.
electrons in degenerate orbitals, i.e., orbitals within the same
22. The maximum orbital angular momentum of an elec-
subshell which have the same values of n and l, states that in
tron with n = 5 is
degenerate orbitals pairing of electrons does not occur unless
h h and until all such orbitals are filled singly with their paral-
(a) 6 (b) 12
2p 2p lel spin. However, degeneracy of orbitals of a given subshell
h h may be destroyed in an inhomogeneous magnetic field like
(c) 42 (d) 20 Octahedral field, tetrahedral field, etc. For example, d-orbital
2p 2p
23. The orbital angular momentum of an electron in p- is five-fold degenerate and degeneracy is destroyed in an ap-
orbital makes an angle of 45° from Z-axis. Hence Z- plied magnetic field provided that it is in homogenous condi-
component of orbital angular momentum of electron is tion. This is called splitting of d-orbital. In an octahedral field
(Oh), splitting of d-orbital takes place such that dxy, and dxz
h Ê hˆ h Ê hˆ
(a) (b) ÁË ˜¯ (c) - (d) - ÁË ˜¯ orbitals (triply degenerate) have lower energy than d x2 - y 2
p 2p p 2p
24. The spin-only magnetic moment of a free ion is 8 and (doubly degenerate) d z 2 which may again split into two
B.M. The spin angular momentum of electron will be levels: d x2 - y 2 and the former having higher energy than the
h h h 3 h latter. The pairing of electron in lower energy may occur if
(a) 2 (b) 8 (c) 6 (d)
2p 2p 2p 4 2p the energy difference between the two level called splitting
Passage 6 energy (D.E.) is greater than the pairing energy, Ep, i.e., the
For a single electron atom or ion, the wave number of radia- increase in energy resulting from interelectronic repulsion
tion emitted during the transition of electron from a higher during pairing so as to yield an electronic configuration of the
energy state (n = n2) to a lower energy state (n = n1) is given lowest possible energy state, i.e., the ground state. This con-
by the expression: dition is met only in the strong magnetic field. In tetrahedral
field (Td), the splitting is just the reverse of the Oh field. A

IIT JEE PC-V1_02.indd 24 3/21/2017 5:15:30 PM


Join our telegram channel: @Jee_neet_topper

Atomic Structure 2.25

spinning electron behaves as though it were a tiny bar magnet (a) Statement I is true, Statement II is true; Statement II is
with poles lying on the axis of spin. The magnetic moment of the correct explanation for Statement I.
any atom, ion or molecule due to spin called spin-only mag- (b) Statement I is true, Statement II is true; Statement II is
netic moment (ms) is given by the formula: NOT a correct explanation for statement-I
m s = n(n + 2) B.M where n = number of unpaired (c) Statement I is true, Statement II is false.
electron(s). (d) Statement I is false, Statement II is true.
27. The spin-only magnetic moment of Cr3+ (EC [Ar] 3d3) 35. Statement I: Nodal plane of px atomic orbital in yz
in strong Oh magnetic field is equal to plane.
Statement II: In px, atomic orbital electron density is
(a) 3 B.M (b) 8 B.M zero in the yz plane.
(c) 15 B.M (d) Zero 36. Statement I: No two electrons in an atom can have the
28. The spin-multiplicity of Fe3+ (Ec[Ar]3d5) in its ground same values of four quantum numbers.
state but placed in a strong Oh magnetic field is Statement II: No two electrons in an atom can be si-
(a) 6 (b) 2 (c) 3 (d) 4 multaneously in the same shell, same subshell, same
29. Statement 1: The kinetic energy of an electron is orbitals and have the same spin.
greater than a-particle having same momentum. 37. Statement I: p-orbital has dumb-bell shape.
Statement 2: Mass of a-particle is greater than the Because
mass of electron. Statement II: Electrons present in p-orbital can have
(a) Statements 1 and 2 are true. Statement 2 is the one of three values for ‘m’, i.e., 0, +1, –1
correct explanation for statement 1. 38. Statement I: The ground state configuration of Cr is
(b) Statements 1 and 2 are true. Statement 2 is NOT 3d5 4s1.
the correct explanation for Statement 1. Statement II: A set of exactly half-filled orbitals con-
(c) Statement 1 is true, Statement 2 is false. taining parallel spin arrangement provide extra stabil-
(d) Statement 1 is false, Statement 2 is true. ity.
30. Statement 1: For hydrogen orbital energy increases as 39. Statement I: Mass numbers of the elements may be-
1s < 2s < 2p < 3s < 3p < 3d < 4s < 4p … fractional.
Statement 2: The orbital with lower (n + l) value has Statement II: Mass numbers are obtained by the sum
lesser energy and hence filled up first. of mass of electron and neutron.
31. Statement 1: When two or more empty orbitals of 40. Statement I: Limiting line in the Balmer series has a
equal energy are available. One electron must be placed wavelength of 36.4 mm.
in each until they are all half-filled. Statement II: Limiting lines is obtained for a jump of
Statement 2: The pairing of electron is an unfavour- electron from n = • to n = 2 for Balmer series.
able phenomenon. 41. Statement I: The electronic configuration of nitrogen
32. Statement 1: The Paschen series in the spectrum of atom is represented as
hydrogen lies in the infra-red region.
Statement 2: The Paschen series is born of transition
not as
of electrons onto the second orbit from higher orbits.
33. Statement 1: The principal quantum number, n, can
have whole number values 1, 2, 3, … etc. Statement II: The configuration of ground state of an
Statement 2: The angular momentum quantum num- atom is the one which has the greatest multiplicity.
ber can never be equal to n. 42. Statement I: The configuration of B atom cannot be
34. Question: Is the specie paramagnetic? 1s2 2s3.
Statement 1: The atomic number of specie is 29. Statement II: Hund’s rule demands that the configura-
Statement 2: The charge on the specie is +1. tion should display maximum multiplicity.
(a) Statement (1) alone is sufficient but statement (2) 43. Statement I: 2p orbitals do not have spherical nodes.
is not sufficient. Statement II: The number of spherical nodes in p-or-
(b) Statement (2) alone is sufficient but statement (1) bitals is given by (n – 2).
is not sufficient. 44. Statement I: In Rutherford’s gold foil experiment,
(c) Both statements together are sufficient but neither very few a-particles are deflected back.
statement alone is sufficient. Statement II: Nucleus present inside the atom is
(d) Statements (1) and (2) together are not sufficient. heavy.
45. Statement I: Each electron in an atom either +½ or –½
(Assersion and Reason)
spin quantum number
These questions contains, Statement I (assertion) and State- Statement II: Spin quantum numbers are obtained by
ment II (reason). Bohr’s Theory.

IIT JEE PC-V1_02.indd 25 3/21/2017 5:15:31 PM


Join our telegram channel: @Jee_neet_topper

2.26 Physical Chemistry-I for JEE (Main & Advanced)

46. Statement I: There are two spherical nodes in 3s-orbit- 58. What is the number of waves made by a Bohr electron
al. in Hydrogen atom in one complete revolution in the
Statement II: There is no angular node in 3s-orbital. third orbit?
47. Choose the correct relation on the basis of Bohr’s 59. The number of nodal plane in dxy orbital.
theory. 60. What is the number of nodes in the radial distribution
1 curve of 2s orbital?
(a) Velocity of electron µ
n 61. Magnetic moment of Mx+ is 24 BM . What is the
Z2 number of unpaired electrons in Mx+?
(b) Frequency of revolution µ 3
n 62. The wave function of an orbital is represented as y420.
(c) Radius of orbit µ n2Z What is the azimuthal quantum number of that orbital?
Z3 63. The number of concentric spherical surfaces for 3s
(d) Force on electron µ 4 orbital at which the probability of finding electron is
n
zero, are
48. A hydrogen - like atom has ground state binding energy
64. What is the number of electrons with l = 2 in the ground
122.4 eV. Then
state of Chromium atom?
(a) its atomic number is 3.
65. What is the value of ‘n’ of the highest excited state that
(b) an electron of 90 eV can excite it to a higher state.
an electron of hydrogen atom in the ground state can
(c) an 80 eV electron cannot excite it to a higher state.
reach when 12.09 eV energy is given to the hydrogen
(d) an electron of 8.2 eV and a photon of 91.8 eV are
atom?
emitted when a 100 eV electron interacts with it.
66. A transition metal cation M+3 has magnetic moment
49. Energy equivalent of 10.00 cm–1 is –
(a) 2 ¥ 10–22 J per photon 35 BM . What is the number of unpaired electrons in
(b) 2.9 ¥ 10–2 kcal mol–1 photon ns orbital of M+3?
(c) 1.2 ¥ 10–1 kJ mol–1 photon (Note: Transition metal exhibit ferromagnetic proper-
(d) 2 ¥ 10–15 ergs per photon ty)
50. Radiation of wavelength 200 Å falls on a platinum sur- 67. The circumference of the second, orbit of electron in
face. If the work function of the metal is 5 eV, which of hydrogen atom is 400 nm, the de Broglie wavelength
the following results are correct about experiment? of electron corresponding to the circumference of same
(a) The velocity of photoelectrons increases with in- orbit is 200 nm. What is the number of waves made by
crease in intensity of radiation. an electron?
(b) Photo-emission of electrons takes place. 68. What is the number of waves made by a Bohr electron
(c) The threshold frequency of the metal is 1.21 ¥ in an orbit of maximum magnetic quantum 3?
1015 sec–1. 69. How many elements does the last electron have the
(d) The velocity of the photo-electrons is 4.48 ¥ quantum numbers of n = 4 and l = 1?
106 m/sec. 70. What is the orbital angular momentum of an electron in
51. Which of the following statement (s) is/are correct? 3s – orbital?
(a) Fe3+ and Mn2+ have equal paramagnetic character. 71. How many d – electrons in Cu+(At.No = 29) can have
(b) Cu2Cl2 and CuCl2 are coloured. Ê 1ˆ
the spin quantum Á - ˜ ?
(c) MnO -4 is purple in colour because of unpair d Ë 2¯
electrons. 72. What is the maximum number of electrons in an atom
(d) The magnetic moment of Fe2+ and Co3+ is equal to that can have the quantum numbers n = 4, me = ±1?
2 6 B.M. 73. What is the number of radial nodes for 3P – orbital?
52. What is the number of waves made by an elelctron 74. How long would it take a radio wave of frequency
moving in an orbit having maximum magnetic quan- 6 ¥ 103 sec–1 to travel from Mars to Earth, a distance
tum number +3? of 8 ¥ 107 km?
53. What is the number of spectral lines produced when an 75. The energy levels of hypothetical one electron atom are
electron jumps from fifth orbit to second orbit in the shown below.
hydrogen atom? 0 eV —— n = •
54. What is the orbital angular momentum of 2s electron? –0.50 eV —— n = 5
h –1.45 eV —— n = 4
55. The angular momentum of an electron is . The –3.08 eV —— n = 3
p
–5.3 eV —— n = 2
Bohr’s orbit in which the electron is revolving is.
–15.6 eV —— n = 1
56. What is the the number of radial nodes of 2p orbital?
(a) Find the ionisation potential of atom?
57. An oil drop has –6.39 ¥ 10–19 Coulomb charge. What is
(b) Find the short wavelength limit of the series ter-
the number of electrons in the oil drop?
minating at n = 2?

IIT JEE PC-V1_02.indd 26 3/21/2017 5:15:32 PM


Join our telegram channel: @Jee_neet_topper

Atomic Structure 2.27

(c) Find the wave number of photon emitted for the 89. Estimate the difference in energy between I and II Bohr
transition made by the electron from third orbit to orbit for a hydrogen atom. At what minimum atomic
first orbit? number a transition from n = 2 to n = 1 energy level
(d) Find the minimum energy that an electron will would result in the emission of X-rays with l = 3.0 ¥
have after interacting with this atom in the ground 10–8 m? Which hydrogen-like species does this atomic
state, if the initial kinetic energy of the electron is number correspond to?
(i) 6 eV and (ii) 11 eV? 90. 1.8 g atoms of hydrogen are excited to radiations. The
76. Suppose 10–17 J of light energy is needed by the interior study of spectra indicates that 27% of the atoms are in
of the human eye to see an object. How many photons third energy level and 15% of atoms in second energy
of green light (l = 550 nm) are needed to generate this level and the rest in ground state. If I.P. of H is 21.7 ¥
minimum amount of energy? 10–12 erg. Calculate
77. Find the number of photons of radiation of frequency (i) number of atoms present in third and second en-
5 ¥ 1013 s–1 that must be absorbed in order to melt one ergy level.
g ice when the latent heat of fusion of ice is 330 J/g. (ii) total energy evolved when all the atoms return to
78. The eyes of certain member of the reptile family pass ground state.
a single visual signal to the brain when the visual re- 91. One mole He+ ions are excited. Spectral analysis
ceptors are struck by photons of wavelength 850 nm. showed existence of 50% ions in 3rd orbit, 25% in 2nd
If the total energy of 3.15 ¥ 10–14 J is required to trip and rest in ground state. Calculate total energy evolved
the signal, what is the minimum number of photons when all the ions return to the ground state in terms of
that must strike the receptor? NA and DE.
79. The wavelength of a certain line in the Paschen series 92. The energy of an excited H-atom is –3.4 eV. Calculate
is 1093.6 nm. What is the value of nhigh for this line angular momentum of e–.
[RH = 1.0973 ¥ 10+7 m–1]? 93. The vapours of Hg absorb some electrons accelerated
80. Wavelength of the Balmer Ha line (first line) is 6565 Å. by a potential difference of 4.5 volt. As a result of it,
Calculate the wavelength of Hb (second line). light is emitted. If the full energy of single incident e– is
81. Calculate the Rydberg constant R if He+ ions are known to be converted into light emitted by single Hg atom,
to have the wavelength difference between the first (of find the wave number of the light.
the longest wavelength) lines of Balmer and Lyman se- 94. The hydrogen atom in the ground state is excited by
ries equal to 133.7 nm. means of monochromatic radiation of wavelength x A0.
82. Calculate the energy emitted when electrons of 1.0 g The resulting spectrum consists of 15 different lines.
atom of hydrogen undergo transition giving the spec- Calculate the value of x.
tral line of lowest energy in the visible region of its 95. A plot of P(r) of 1s electron of H-atom versus distance
atomic spectrum. from the nucleus (r) is shown in the figure given below.
83. A photon having l = 854 Å causes ionisation of a nitro- Find the value of r corresponding to the maximum in
gen atom. Give the I.E. per mole of nitrogen in KJ. terms of a0 where the maximum occurs.
84. Calculate energy of electron which is moving in the or- Given:
bit that has its radius, 16 times the radius of first Bohr Ê 1 ˆ
1/2

orbit for H-atom. P(r ) = 4p r 2y 2 y 1s (for H-atom) = Á 3 ˜ e – r / a0


Ë p a0 ¯
85. The electron energy in hydrogen atom is given by
– 21.7 ¥ 10 –12
En = ergs. Calculate the energy required
n2 P(r)

to remove an e completely from n = 2 orbit. What is r
the largest wavelength in cm of light that can be used
96. Absorption of energy by an atom of hydrogen in the
to cause this transition?
ground state results in the ejection of electron with
86. Calculate the wavelength in angstrom of photon that
de-Broglie wavelength l = 4.7 ¥ 10–10 m. Given that
is emitted when an e– in Bohr orbit n = 2 returns to the
the ionisation energy is 13.6 eV, calculate the energy of
orbit n = 1. The ionisation potential of the ground state
photon which caused the ejection of electron.
of hydrogen atom is 2.17 ¥ 10–11 erg/atom.
97. Photo-chemical dissociation of oxygen results in the
87. The velocity of e– in a certain Bohr orbit of the hydro-
production of two oxygen atoms, one in the ground
gen atom bears the ratio of 1 : 275 to the velocity of
state and one in the excited state.
light. What is the quantum number “n” of the orbit.
hn
88. A doubly ionised lithium atom is hydrogen like with O 2 ææÆ O + O *
atomic number Z = 3. Find the wavelength of the radia- The maximum wavelength needed for this is 174 nm.
tion required to excite the electron in Li2+ from the first If the excitation energy O Æ O* is 3.15 ¥ 10–19J. How
to the third Bohr orbit.

IIT JEE PC-V1_02.indd 27 3/21/2017 5:15:33 PM


Join our telegram channel: @Jee_neet_topper

2.28 Physical Chemistry-I for JEE (Main & Advanced)

much energy in kJ/mole is needed for the dissociation weight of natural boron is 10.81. Calculate the percent-
of one mole of oxygen into normal atoms in ground age of each isotope in natural boron. [IIT-JEE 1978]
state? 8. The energy of the electron in the second and the third
98. If the average lifetime of an excited state of H atom Bohr’s orbits of the hydrogen atom is –5.42 ¥ 10–12 erg
is of order 10–8 sec, estimate how many orbits an e– and –2.41 ¥ 10–12 erg respectively. Calculate the wave-
makes when it is in the state n = 2 and before it suffers length of the emitted radiation when the electron drops
a transition to n = 1 state. from the third to the second orbit. [IIT-JEE 1981]
99. Calculate the frequency of e– in the first Bohr orbit in a 9. Calculate the wavelength in Angstroms of the pho-
H-atom. ton that is emitted when an electron in the Bohr orbit,
100. A single electron orbits around a stationary nucleus of n = 2 returns to the orbit, n = 1 in the hydrogen atom.
charge +Ze where Z is a constant from the nucleus and The ionisation potential of the ground state hydrogen
atom is 2.17 ¥ 10–11 erg per atom. [IIT-JEE 1982]
e is the magnitude of the electric charge. The hydro-
10. The electron energy in hydrogen atom is given by
gen like species required 47.2 eV to excite the electron
E = (–21.7 ¥ 10–12)/n2 ergs. (a) Calculate the energy
from the second Bohr orbit to the third Bohr orbit. Find
required to remove an electron completely from the
the value of Z give the hydrogen like species formed.
n = 2 orbit. What is the longest wavelength (in cm) of
101. A stationary He+ ion emitted a photon corresponding to light that can be used to cause this transition?
a first line of the Lyman series. The photon liberated a [IIT-JEE 1984]
photoelectron from a stationary H atom in ground state. 11. What is the maximum number of electrons that my be
What is the velocity of photoelectron ? present in all the atomic orbitals with principle quan-
102. Calculate the threshold frequency of metal if the bind- tum number 3 and azimuthal quantum number 2?
ing energy is 180.69 kJ mol–1 of electron. [IIT-JEE 1985]
103. Calculate the binding energy per mole when threshold 12. According to Bohr’s theory, the electronic energy
wavelength of photon is 240 nm. of hydrogen atom in the nth Bohr’s orbit is given by
– 21.76 ¥ 10 –19
En = J . Calculate the longest wave-
PREVIOUS YEARS’ QUESTIONS n2
OF JEE (MAIN & ADVANCED) length of light that will be needed to remove an elec-
1. The increasing order (lowest first) for the values of e/m tron from the third Bohr orbit of the He+ ion.
(charge/mass) for electron (e), proton (p), neutron (n) [IIT-JEE 1990]
and alpha particle (a) is [IIT-JEE 1984] 13. A ball of mass 100 g is moving with 100 ms–1. Find its
(a) e, p, n,a (b) n, p, e,a wavelength. [IIT-JEE 2004]
(c) n, p,a, e (d) n, a, p, e 14. Find the velocity (ms–1) of electron in first Bohr’s orbit
2. The triad of nuclei that is isotonic is [IIT-JEE 1988] of radius a0. Also find the de Broglie’s wavelength (in
14 15 17 12 14 19
m). Find the orbital angular momentum of 2p orbital of
(a) 6 C, 7 N, 9 F (b) 6 C, 7 N, 9 F hydrogen atom in units of h/2p. [IIT-JEE 2005]
(c) 14 14 17
(d) 14 14 19 15. Estimate the difference in energy between first and sec-
6 C, 7 N, 9 F 6 C, 7 N, 9 F
ond Bohr orbit for a hydrogen atom. At what minimum
3. Rutherford’s experiment on scattering of particles
atomic number, a transition from n = 2 to n = 1 energy
showed for the first time that the atom has
level would result in the emission of X-rays with = 3.0
[IIT-JEE 1981]
¥ 10–8 m? Which hydrogen atom-like species does this
(a) electrons (b) protons
atomic number correspond to? [IIT-JEE 1993]
(c) nucleus (d) neutrons
16. What transition in the hydrogen spectrum would have
4. The wavelength associated with a golf ball weighing
the same wavelength as the Balmer transition n = 4 to n
200 g and moving at a speed of 5 m/h is of the order
= 2 of He+ spectrum? [IIT-JEE 1993]
[IIT-JEE 2000]
17. Find out the number of waves made by a Bohr electron
(a) 10–10m (b) 10–20 m (c) 10–30m (d) 10–40m
in one complete revolution in its third orbit.
5. Correct set of four quantum numbers for the valence
[IIT-JEE 1994]
(outermost) electron of rubidium (Z = 37) is
18. Iodine molecule dissociates into after absorbing light
[IIT-JEE 1984]
of 4500 Å. If one quantum of radiation is absorbed by
(a) 5, 0, 0, + ½ (b) 5, 1, 0, + ½
each molecule, calculate the kinetic energy of iodine
(c) 5, 1, 1, + ½ (d) 6, 0, 0, + ½
atoms. (Bond energy of I2 = 240 kJ mol–1.)
6. The outermost electronic configuration of the most
[IIT-JEE 1995]
electronegative element is [IIT-JEE 1988]
19. Calculate the wave number for the shortest wavelength
(a) ns2np3 (b) ns2np4 (c) ns2np5 (d) ns2np6
transition in the Balmer series of atomic hydrogen.
7. Naturally occurring boron consists of two isotopes
[IIT-JEE 1996]
whose atomic weights are 10.01 and 11.01. The atomic

IIT JEE PC-V1_02.indd 28 3/21/2017 5:15:33 PM


Join our telegram channel: @Jee_neet_topper

Atomic Structure 2.29

20. With what velocity should an a-particle travel towards (D) Probability density (s) Electron spin
the nucleus of a copper atom so as to arrive at a dis- of electron at the quantum number
tance 10–13 metre from the nucleus of a copper atom? nucleus in hydro-
[IIT-JEE 1997] gen – like atom
21. An electron beam can undergo diffraction by crystals.
Through what potential should a beam of electrons be
accelerated so that its wavelength becomes equal to Comprehension-Based Questions
1.54 Å? [IIT-JEE 1997] The hydrogen- like species Li2+ is in a spherically symmetric
22. Calculate the energy required to excite one litre of hy- state S1 with one radial node. Upon absorbing light, the ion
drogen gas at 1 atm and 298 K to the first excited state undergoes transition to a state S2. The state S2 has one radial
of atomic hydrogen. The energy for the dissociation of node and its energy is equal to the ground state energy of the
H-H bond is 436 kJ mol–1. [IIT-JEE 2000] hydrogen atom.
23. Wavelength of high energy transition of H-atoms is 27. The state S1 is (2010)
91.2 nm. Calculate the corresponding wavelength of (a) 1s (b) 2s (c) 2p (d) 3s
He atoms. [IIT-JEE 2003] 28. Energy of the state S1 in units of the hydrogen atom
24. The Schrodinger wave equation for hydrogen atom is ground state energy is (2010)
[IIT-JEE 2004] (a) 0.75 (b) 1.50 (c) 2.25 (d) 4.50
1 Ê 1ˆ
3/2
Ê r0 ˆ – r0 / a0 29. The orbital angular momentum quantum number of the
Y2s =
4 2p ÁË a0 ˜¯ ÁË 2 – a ˜¯ e state S2 is (2010)
0
(a) 0 (b) 1 (c) 2 (d) 3
where a0 is Bohr’s radius. If the radial node in 2s be at
r0, find r0 in terms of a0. Integer-Type Questions
25. Match the columns according the Boh’s theory. 30. The maximum number of electrons that can have prin-
En = Total energy cipal quantum number, n = 3 and spin quantum num-
Kn = Kinetic energy 1
ber, ms = - , is (2011)
Vn = Potential energy 2
rn = Radius of nth orbit (2006, 6M) 31. Which of the following is the energy of a possible ex-
cited state of hydrogen? [2015 Main]
Column I Column II (a) +13.6 eV (b) –6.8 eV
(A) Vn/Kn = ? (p) 0 (c) –3.4 eV (d) +6.8 eV
(B) x (q) –1 32. The correct set of four quantum numbers for the va-
If radius of nth orbit µ En , x = ? lence electrons of rubidium atm (Z = 37) is
(C) Angular momentum in lowest or- (r) –2 [2013 Main]
bital 1 1
(a) 5, 0, 0, + (b) 5, 1, 0, +
1 2 2
(D) µ zy, y = ? (s) 1 1 1
rn (c) 5, 1, 1, + (d) 5, 0, 1, +
2 2
26. Match the entries in column I with the related quantum 33. Energy of an electron is given by E = –2.78 ¥ 10–18 J
number(s) in Column II. (2008, 6M) Ê Z2ˆ
Column I Column II ÁË 2 ˜¯ . Wavelength of light required to excite an elec-
n
(A) Orbital angular (p) Principal quantum tron in an hydrogen atom from level n = 1 to n = 2 will
momentum of the number be [2013 Main]
electron in a hy- (h = 6.62 ¥ 10–34 Js and c = 3.0 ¥ 108 ms–1)
drogen- like atomic (a) 1.214 ¥ 10–7 m (b) 2.816 ¥ 10–7 m
–7
orbital (c) 6.500 ¥ 10 m (d) 8.500 ¥ 10–7 m
34. The kinetic energy of an electron in the second Bohr
(B) A hydrogen – like (q) Azimuthal quan-
orbit of a hydrogen atom is [a0 is Bohr radius]
one electron wave tum number
[2012]
function obeying 2 2
Pauli’s principle h h
(a) (b)
(C) Shape, size and (r) Magnetic quantum 4p 2 ma02 16p 2 ma02
orientation of hy- number h2 h2
drogen- like atomic (c) (d)
orbitals 32p 2 ma02 64p 2 ma02

IIT JEE PC-V1_02.indd 29 3/21/2017 5:15:34 PM


Join our telegram channel: @Jee_neet_topper

2.30 Physical Chemistry-I for JEE (Main & Advanced)

35. In an atom, the total number of electrons having quan- 36. The atomic masses of He and Ne are 4 and 20 amu,
1 respectively. The value of the de-Broglie wavelength
tum numbers n = 4, |m| = 1 and m2 = - is of He gas at –73°C is ‘M’ times that of the de-Broglie
2
[2014 Adv.] wavelength of Ne at 727°C M is [2013 Adv.]

Answer Key

LEVEL I

1. (a) 2. (c) 3. (d) 4. (c) 5. (a) 6. (d) 7. (b) 8. (d) 9. (a) 10. (d)
11. (d) 12. (d) 13. (b) 14. (a) 15. (b) 16. (b) 17. (c) 18. (b) 19. (b) 20. (c)
21. (d) 22. (a) 23. (c) 24. (b) 25. (b) 26. (b) 27. (a) 28. (a) 29. (b) 30. (c)
31. (c) 32. (d) 33. (c) 34. (c) 35. (a) 36. (a) 37. (c) 38. (a) 39. (a) 40. (b)
41. (b) 42. (a) 43. (a) 44. (a) 45. (c) 46. (c) 47. (d) 48. (b) 49. (a) 50. (c)
51. (c) 52. (c) 53. (b) 54. (c) 55. (a) 56. (a) 57. (d) 58. (b) 59. (c) 60. (c)
61. (d) 62. (b) 63. (c) 64. (a) 65. (c) 66. (a) 67. (b) 68. (d) 69. (b) 70. (b)
71. (a) 72. (c) 73. (a) 74. (b) 75. (a) 76. (c) 77. (c) 78. (a) 79. (b) 80. (b)
81. (b) 82. (d) 83. (b) 84. (b) 85. (d) 86. (c) 87. (a) 88. (b) 89. (b) 90. (a)
91. (a) 92. (a) 93. (a) 94. (b) 95. (d) 96. (d) 97. (d) 98. (a) 99. (a) 100. (b)
101. (a) 102. (b) 103. (a) 104. (c) 105. (d) 106. (b) 107. (d) 108. (b) 109. (d) 110. (c)
111. (c) 112. (d) 113. (b) 114. (b) 115. (c) 116. (c) 117. (a) 118. (b) 119. (a) 120. (a)
121. (c) 122. (d) 123. (a) 124. (d) 125. (b) 126. (d) 127. (b) 128. (a) 129. (d) 130. (a)
131. (a) 132. (a) 133. (a) 134. (b) 135. (b) 136. (c) 137. (a) 138. (b) 139. (c) 140. (d)
141. (b) 142. (c) 143. (d) 144. (a) 145. (c) 146. (c) 147. (c) 148. (c) 149. (c) 150. (d)

LEVEL II

1. (d) 2. (d) 3. (a) 4. (a) 5. (b) 6. (a) 7. (b) 8. (a) 9. (a) 10. (d)
11. (a) 12. (d) 13. (b) 14. (c) 15. (a) 16. (b) 17. (c) 18. (a) 19. (c) 20. (b)
21. (a) 22. (a) 23. (d) 24. (d) 25. (b) 26. (b) 27. (a) 28. (b) 29. (d) 30. (b)
31. (a) 32. (c) 33. (d) 34. (a) 35. (b) 36. (d) 37. (a) 38. (d) 39. (d) 40. (b)
41. (d) 42. (c)

LEVEL III

1. (A) Æ P; (B) Æ P, Q, R, S, T; (C) Æ Q, R, S; (D) Æ P 2. A Æ r, t; B Æ p; C Æ q, s; D Æ q


3. A Æ p; B Æ p; C Æ p, q, r, s; D Æ p 4. A Æ r, p; B Æ r, t; C Æ s; D Æ q
5. A Æ r; B Æ q; C Æ p, t; D Æ s 6. (A) Æ t; (B) Æ s ; (C) Æ u ; (D) Æ q ; (E) Æ p; (F) Æ r
7. (A) Æ u ; (B) Æ s ; (C) Æ p ; (D) Æ t ; (E) Æ q ; (F) Æ r
8. (A) Æ q ; (B) Æ p ; (C) Æ q, r ; (D) Æ r, s
9. (A) Æ r ; (B) Æ s; (C) Æ p; (D) Æ q
10. A Æ P, BÆp, q, s, CÆp, r, D Æ q, s
11. (b) 12. (b) 13. (c) 14. (b) 15. (a) 16. (c) 17. (d) 18. (d) 19. (d) 20. (b)
21. (d) 22. (d) 23. (b) 24. (a) 25. (a) 26. (b) 27. (c) 28. (b) 29. (a) 30. (d)
31. (c) 32. (c) 33. (b) 34. (c) 35. (a) 36. (a) 37. (b) 38. (a) 39. (e) 40. (a)
41. (a) 42. (b) 43. (a) 44. (b) 45. (e)
46. (b) 47. (a, b, d) 48. (a, c, d) 49. (a, b, c, d) 50. (b, c, d)
51. (a, d) 52. (4) 53. (6) 54. (0) 55. (2) 56. (0) 57. (4) 58. (3) 59. (2) 60. (1)
61. (4) 62. (2) 63. (2) 64. (5) 65. (3) 66. (0) 67. (2) 68. (4) 69. (6) 70. (0)
71. (5) 72. (12) 73. (1) 74. (2.66 ¥ 102 sec.)

IIT JEE PC-V1_02.indd 30 3/21/2017 5:15:35 PM


Join our telegram channel: @Jee_neet_topper

Atomic Structure 2.31

75.(a) [15.6] (b) [233.9 nm] (c) [1.808 ¥ 107 m–1] (d) [–9.6 eV and –4.6 eV] 76. [28] 77. [1022]
5
78.[1.35 ¥ 10 ] 79. [6] 80. [4863 Å] 81. [1.096 ¥ 10 m ] 82. [1.827 ¥ 105J/m]
7 –1

83.[1400 KJ/mole] 84. [–0.85 eV] 85. [5.425 ¥ 10–12 ergs. 3.66 ¥ 10–5cm] 86. [1220 Å]
87.[2] 88. [108.8 eV] 89. [10.2 eV, He] 90. [0.486 NA, 0.27 NA, 56.65 ¥ 1011 erg]
h
91. [0.5 NA (E2 – E1) + 0.25 NA(E2 – E1)] 92. 93. [0.0036 nm–1] 94. 932 Å 95. r = a0
p
96. [3.271 ¥ 10–18 J] 97. [498.3 KJ/mole] 98. [8 ¥ 106] 99. [2.3 ¥ 1013Hz] 100. [5]
6 –34
101. [3.09 ¥ 10 m/sec.] 102. [v0 = 6.626 ¥ 10 ] 103. 318.9 Kj/mole]

PREVIOUS YEARS’ QUESTIONS OF JEE (MAIN & ADVANCED)

1. (d) 2. (a) 3. (c) 4. (c) 5. (a) 6. (c) 7. [B – 11.01 80%, 20% and B 10.01]
8. [6603 Å] 9. [12220 Å] 10. [3.67 ¥ 10–5 cm] 11. (10) 12. [2055 Å]
h
13. [6.62 ¥ 10–25 Å] 14. [2.18 ¥ 106 m sec–1, 3.3 Å, 2 ] 15. [He+] 16. [n Æ 2 to n Æ 1] 17. [3]
p
18. [2.16 ¥ 10–20J] 19. 27419.25 cm–1] 20. [6.3 ¥ 106m sec–1] 21. [63.3 volt] 22. [98.18 KJ]
23. [22.8 nm] 24. [r0 = 2a0] 25. A Æ r; B Æ q, C Æ p, D Æ s
26. A Æ q; B Æ p, q, r, s; C Æ p, q, r ; D Æ p, q, r 27. (b) 28. (c) 29. (b) 30. (9) 31. (c)
32. (a) 33. (a) 34. (c) 35. (6) 36. (5)

Hints and Solutions

LEVEL I 8. (d) d7 =
1 1 1 3
1 Total spin = + + + =
2 2 2 2
1. (a) E5 = –13.6 ¥ = – 0.54 eV
(5) 2 9. (a) K = 2e– = 1s2
2. (c) Li+2 and He+ both have same number of electron so L = 8e– = 2s2 2p4
spectrum pattern will be similar. M = 11e– = 3s2 3p6 3d3
h 1 N = 2e– = 4s2
3. (d) l = lµ l = 2 for d \ e– = 3
2mqV V
10. (d) Cl– = 1s2 2s2 2p6 3s2 3p6
l1 V 200 2 For last e– n = 3, l = 1, m = ±1 or 0 or –1
= 2 = =
l2 V1 50 1
13.6 z 2
h 11. (d) En = - eV
4. (c) Dx ◊ Dp = n2
4p as move away from the nucleus, the energy increases,
put value Dp = 1.0 ¥ 10–5 kg ms–1 hence, energy is maximum at infinite distance from the
5. (a) Orbital angular momentum nucleus.
h 12. (d) When an electron jump from higher level to lower
= l(l + 1) ◊ for l = 0 level, it emits the photon and lower level to higher lev-
2p
el. It absorbs photon. Hence, ‘1s’ only absorbs photon
6. (d) 25Mn = 1s2, 2s2 2p6, 3s2 3p6 3d5, 4s2 because it is lowest energy level.
Mn+4 = 1s2, 2s2 2p6, 3s2 3p6 3d3, 4s0 1 È1 1˘
7. (b) 30Zn2+ = 1s2, 2s2 2p6, 3s2 3p6 3d10 13. (b) = RH Í 2 – 2 ˙
l Î n1 n2 ˚
(unpaired de– = 0)
2+ 2 2 6 2 6 6 – In Balmer series, electron jumps from higher energy
26Fe = 1s 2s 2p 3s 3p 3d (unpaired de = 4)
3+ 2 2 6 2 6 7 –
level to second energy level. Hence third line form
28Ni = 1s 2s 2p 3s 3p 3d (unpaired de = 3) when electrons jump from fifth energy level to 2
+ 2 2 6 2 6 9 –
29Cu = 1s 2s 2p 3s 3p 3d (unpaired de = 1) energy level. 5 Æ 2

IIT JEE PC-V1_02.indd 31 3/21/2017 5:15:36 PM


Join our telegram channel: @Jee_neet_topper

2.32 Physical Chemistry-I for JEE (Main & Advanced)

14. (a) 37Rb = 1s2 2s2 2p6 3s2 3p6 3d10 4s2 4p6 5s1 1 È1 1 ˘ 1 È 7 ˘
n l m s = RH z 2 Í – ˙ = RH 32 Í ˙
l Î 9 16 ˚ l Î 9 ¥ 16 ˚
5 0 0 + 1/2
15. (b) Aufbau’s principle : electron fills in orbital increas- 9 ¥ 16 1 16
l= ¥ fi l = x
ing order of energy level. 9 ¥ 7 RH 7
70 2+
16. (b) 30 Zn = n = A – Z = 70 – 30 = 40 25. (b) (IE) Li2+ = (IE) H ¥ Z 2
17. (c) n > l, m = –l to + l
= 21.8 ¥ 10–19 ¥ 9 J/atom
n l s
\ Kinetic energy of ejected electron
3 2 ½
= 8 ¥ 21.8 ¥ 10–19 J atom
The value of (m) is wrong h h
l = 2, m = –2, –1, 0, +1, +2 l= =
2KE ◊ m 2qV ◊ m
18. (b) Hund’s rule
19. (b) Cr = 1s2 2s2 2p6 3s2 3p6 3d5 4s1 ; Mn+ = 1s2 2s2 2p6 6.62 ¥ 10 –34
l=
3s2 3p6 3d5 4s1 2 ¥ 9.1 ¥ 10 –31 ¥ 2.18 ¥ 10 –19 ¥ 8
i.e., it represents both ground state and cationic form.
l =1.17 Å
20. (c) Fe3+ = 1s2 2s2 2p6 3s2 3p6 3d5 26. (b) Fe2+ = 1s2 2s2 2p6 3s2 3p6 3d6
21. (d) Schrödinger’s equation gives only n, l and m quan-
tum number. Spin quantum number is not related to
Schrodinger equation. Unpaired electron (n) = 4
1 Magnatic moment
22. (a) hn = hn0 + mn2
2 = n(n + 2) BM = 4(6) = 24 BM
hc hc 1 2 Orbital angular momentum
= + mn
l l0 2 = l ¥ (l + 1)  = 2(3)  fi 6 
Ê l – lˆ 1 2
K.E. = hc Á 0 = mn h 1
Ë l l0 ˜¯ 2 27. (a) l = lµ
2 KE ◊ m 2qV ◊ m
Ê h2 ˆ Ê l0 – l ˆ Ê h ˆ 1 1
Á ˜ = hc ÁË ll ˜¯ \l = le µ ; l p+ =
Ë 2mle2 ¯ 0 ËÁ 2mK ◊ E ˜¯ M e ¥ 16 E M p ¥ 4E
l l0 h
le2 = la µ
1
; hence le > l p + = lµ
[l0 – l ]2mc
4M p ¥ 4 E
1
È h l l0 ˘ 2 28. (a) Li (g) h Li+ + e– ; DH = 520
le = Í ˙
Î 2mc [l0 – l ] ˚ Li+ (g) h Li+2 + e ; DH = a KJ/mol
23. (c) mn = mass of neutron; mp = mass of proton Li2+ (g) h Li2+ + e ; DH = b KJ/mol
mn b = ( IE2 ) Li+ and (IE) Li2+ = (IE)n ¥ z2 = 1313 ¥ 9
2mp
2 b = ( IE2 ) Li+ = 11817 KJ/mol
atomic mass fi (mn + mp) [mn ~ mp] 520 + a + 11817 = 19800
fi (8 + 6) = 14 mp ( IE2 ) Li+ = a = 7463 KJ/mol
new atomic mass fi (4 + 12) = 16 mp
1 Ê 1 1ˆ Ê n2 – n2 ˆ
16 – 14 29. (b) = RH Á 2 – 2 ˜ fi RH Á 2 2 21 ˜
% increase = ¥ 100 = 14.28 % l
14 Ë n1 n2 ¯ Ë n1 n2 ¯
1 È1 1˘ (n22 n12 ) 1
24. (b) = RH Z 2 Í 2 – 2 ˙ l= 2 2
¥
l Î n1 n2 ˚ (n2 – n1 ) RH
for shortest wavelength n2 = •, n1 = 2 First line of Lymen series n2= 2, n1= 1
Second line of Lymen series n2 = 3, n1= 1
1 È1 1 ˘ 4 1
= RH 22 Í – ˙ l = = =x Third line of Lymen series n2= 4, n1 = 1
l Î4 •˚ 4 RH RH 30. (c) The anode ray/canal ray is independent to the elec-
for longest wavelength of Parchan series n2 = 4, n1 = 3 trode material.

IIT JEE PC-V1_02.indd 32 3/21/2017 5:15:39 PM


Join our telegram channel: @Jee_neet_topper

Atomic Structure 2.33

31. (c) Energy order decide from (n + l) rule; (n + l) is mP 1 1 4 v


minimum, energy is minimum. If (n + l) value is equal, = ; or = ◊ a
ma 4 2 1 vp
lower the value of ‘n’, lower the energy.
e3 > e2 > e4 > e1 vp 8 h h
= ; or l = =
r r va 1 p 2mKE
32. (d) r1 = 22 r1 = ; r3 = r1 ¥ n2
n 4 Now,
r3 = r/4 ¥ 9 r3 = 2.25R h
h 6.62 ¥ 10 –34 ¥ 3600 lp 1 2m p ◊ KE p Ê 1ˆ
2
2m ◊ KEa 1
33. (c) l = = = = ; or Á ˜ = a =
mv 0.2 ¥ 5 la 2 h Ë 2¯ 2m p ◊ KE p 4
l = 2.38 ¥ 10–30 metre 2ma ◊ KEa
34. (c) According to Paulis, an orbital can accomdate max-
imum two electrons. Hence Paulis exclusion principle 4 K.E.a 1 K.E. p 16
or, ◊ = ; \ =
violates. 1 K.E. p 4 K.E.a 1
35. Cu(29) = 1s2 2s2 2p6 3s2 3p6 4s1 3d10
44. 1s2 2s2 2p6 3s2 3p6 3d10 4s1
Form (m = 0) = 2 + 2 + 2 + 2 + 2 + 1 + 2 (n + l) = 3 means
= 13 electron 2p6 and 3S2
+2
36. Cd [z = 48] l = 2 means 3d10
Azimuthal l = 1 only for p-orbital n + l = 3 fi 8 electron
upto 48 Æ 2p6 3p6 4p6 = 18e l = 2 fi 10 electron
37. A h 30 W EA (photon) = 13.6 eV 8 ¥ 10 = 80
B h 40 W EB (photon) = 13.6 ¥ 22 = 4 ¥ 13.6 eV 45. Ga last e–1 in 4P1
For A(13.6 ¥ 1.6 ¥ 10–19) ¥ nA = 30 \ n = 4, l = 1
m = –1 or O or +1
For B (4 ¥ 13.6 ¥ 1.6 ¥ 10–19) ¥ nB = 40
S = +½ or –½
nA 3 or single e– system.
=
4 nB 4 46. Bohr’s theory is applicable to H and H-like species.
nA 3 47. Conceptual
fi = Z
nB 1 48. Vn = 2.18 ¥ 108 ¥ cms -1
38. n2 = 11 higher state n
n1 = 1 lowest state for Vn to be maximum; for hydrogen atom
U.V. = 11 – 1 = 10 Z = 1, n = 1 \ V = 2.18 ¥ 108 cms–1
Visible = 11 – 2 = 9 1
11 ¥ 10 49. We know that v µ and n = principal quantum num-
Remaining = – 19 = 55 – 19 n
2 ber. Therefore, vn will be independent of the principal
(z) I.R. = 36 quantum number.
Hence, z – (x + y) = 36 – (9 +10) = 17 50. –P.E. = 2K.E. = 2 ¥ (+13.6) = 27.2 eV.
39. Number of angular nodes = l = 2 (d-orbitals) \ P.E. = –27.2 eV
and number of maxima = n – l = 3 fi n = 5 P.E. with respect to ground state = 27.2 eV, which we
40. 2prn = nl assume to be zero.
4h2 13.6
2p r4 2p r2 K.E. = - 2 eV
l4 = l2 = n
4 2 For the first excited state, n = 2.
È 0.529(4) 2 0.529(2) 2 ˘ 13.6
l4 - l2 = 2p Í - ˙Å \ K.E. = - 2 eV = - 3.4 eV
Î 1¥ 4 2 ˚ 2
= 2p (0.529) ¥ 2 Total energy = (27.2 –3.4)eV = 23.8eV.
= 4pr1 = 6.64 Å 2p rn 2p r1 ¥ n 2
51. T = = or T µ n3; n = 2 here
h un u1 / n
l Ê 1 ˆ mp ◊ vp m v
41. P = Á ˜ = = a a 1
la Ë 2 ¯ h mp ◊ vp 52. DE = E2 - E1; E2 - E1 or l µ
( E2 - E1 )
ma va

IIT JEE PC-V1_02.indd 33 3/21/2017 5:15:40 PM


Join our telegram channel: @Jee_neet_topper

2.34 Physical Chemistry-I for JEE (Main & Advanced)

53. N = number of lines emitted 74. According to de Broglie


diff. ¥ (diff. + 1) 3(3 + 1) 12 h 6.62 ¥ 10-27 erg sec
= = = =6 l= =
2 2 2 mv 2
¥ 5 ¥ 104 cm/sec
54. hn1 = hn0 + K.E.1 6.023 ¥ 1023
2hn1 = hn0 + K.E.1
K.E. = hn1 – hn0 = 4 ¥ 10-8 cm = 4Å
The value of K.E. will increase but more than double of 75. Z = 20, so electronic configuration is
the previous K.E. 1s2 2s2 2p6 3s2 3p6 4s2
55. Aufbau’s principle
56. For outermost electron of Rb, the configuration is 5s1. \ for outer most electron 4s electron n = 4, l = 0, m =
57. The electronic configuration of sulphur atoms 1s2s22p- 1
6 2 4
0, s = +
3s 3p , the orbitals with (n + l) = 3 are 2p and 3s, 2
therefore 8 electrons in sulphur atom have (n + l) = 3. 76. The electronic configuration of Ar atom is 1s2 2s2 2p6
58. Orbital angular momentum 3s2 3p6. In its first excitation state, the electron from 3p
h h will unpair and shift to 4s where its quantum numbers
= l (l + 1) = 0(0 + 1) =0
2p 2p will be 4, 0, 0, ± ½.
59. Number of unpaired electrons in V, Cr and Mn is 3, 6 77. Most electro-negative element is F.
and 5. \ electronic configuration: ns2np5
60. Double dumb bell shaped in 4th principle quantum 78. Angular nodes are only associated with directional or-
number means 4d
bital.
n–l–1=4–2–1=1
E 22 4
2 1 1 79. E2(He+ ) = 21 = -13.6 ¥ 2 = -13.6 ¥ = -13.6 eV
61. Charge/mass for n = 0, a = , p = , e = n n 4
4 1 1/1837
So, order is n, a, p, e. Z2
80. En = -13.6 ¥
62. Charge/mass for n2
1 En (H)
n = 0, a = 2/4, p = 1/1 and e = 1 1
1/1837 = = = 1: 4
En (Be+3 ) 42 4
63. Number of p, n and e are 20, 20 and 20.
64. Isotonic : same number of neutrons 4
65. Conceptual 3 5 27
81. Ratio = E2 – E1/E3 – E2 = =
4 36 5
n2
66. r = r0 = 0.529 ¥ 10-8 cm
Z 1 È1 1˘
82. = RH Z 2 Í 2 - 2 ˙ for Balmer series
Radius of second Bohr orbit 0.529 22 l Î n1 n2 ˚
67. = ¥ =4
Radius of first Bohr orbit 0.529 1 1 È1 1˘ R 5
= RH ¥ 12 Í 2 - 2 ˙ = H cm -1
3+ 0.529 ¥ (2) 2 l Î2 3 ˚ 36
68. rBe = = 0.529
4 1 È1 1˘
69. Absorption line in the spectra arise when energy is ab- 83. = RH ¥ 12 Í 2 - 2 ˙ …(A)
lH n
Î 1 n2˚
sorbed, i.e., electron shifts from lower to higher orbit.
Maximum wavelength means minimum energy so, the 1 È1 1˘
transition involving minimum energy is the right op- = RH ¥ Z 2 Í 2 - 2 ˙ …(B)
lX Î n1 n2 ˚
tion.
70. Conceptual Dividing (A) by (B), we get
lX 1
È1 1˘ = 2
71. VH = RH Í 2 – 2 ˙ ; Z =1 for H-atom lH Z
Î2 3 ˚
1 1
È1 1˘ =
VBe3+ = RH ¥ 16 Í 2 – 2 ˙ ; Z = 4 for Be3+ 4 Z2
Î2 3 ˚
Z = 2 (He+)
Thus, V Be3+ = 16V H = 16 ¥ 15200 cm
84. for hydrogen
= 2.432 ¥ 105 cm–1.
h 6.6 ¥ 10-34 Ê1 1 ˆ Ê 1 1ˆ
72. l = = = 2.37 ¥ 10-30 m Z H2 ¥ Á - 2 ˜ = Z X2ÁË 2 - 2 ˜¯
mv (0.2) ¥ 5/(60 ¥ 60) Ë1 3 ¯ 3 9
h sixth line of Paschen series = 9
73. l = l will be large if m is small.
mv

IIT JEE PC-V1_02.indd 34 3/21/2017 5:15:42 PM


Join our telegram channel: @Jee_neet_topper

Atomic Structure 2.35

8 Ê1 1 ˆ hc
= Z X2 Á - ˜ 92. E =
9 Ë 9 81¯ l
8 8 1
= Z2 fi lµ
9 81 E
Z2 = 9 l1 E2 4000 Å
\ = =
Z = 3 (Li+2) l2 E1 2000 Å
85. hn = hn0 zero velocity K.E. = 0 \ l1 :l2 = 2 : 1
hn = hn0 93. For a ‘d’ electron, l = 2
hn = 4 eV Orbital angular momentum
hc h h
= 4 eV = l (l + 1) = 6 or 6
l 2p 2p
h ¥ c 6.626 ¥ 10-34 ¥ 3 ¥ 108 12
l= = = 3100 Å 94. E = -13.6 eV ¥ = - 3.4 eV
4 eV 4 ¥ 1.6 ¥ 10-19 22
h h
86. l = K.E. = eV K.E. = eV = qV 95. DP ◊ Dx ≥
2mKE 4p
same mass 96. n = 4
l=2
l1 KE 2 2
= = m = –2, –1, 0, +1, +2
l2 KE1 1 1
s=±
\ l1 = 2 l2 2
h 6.626 ¥ 10-34 1 Ê 1 1ˆ
87. l = = 97. = RH (1) 2 Á 2 - 2 ˜
mV 9.1 ¥ 10-31 ¥ 2.2 ¥ 106 l Ë 2 3 ¯
= 0.33 ¥ 10–9 Ê 5ˆ Ê 5R ˆ
= RH ◊ Á ˜ cm -1 or n = Á H ˜ cm -1
= 0.33 nm Ë 36 ¯ Ë 36 ¯
h 6.626 ¥ 10-34 98. For Balmer series n2 = 2
88. l = =
mV 0.200 ¥ 10-3 ¥ 5/3600 Fourth line is 6 to 2
22 -13.6 ¥ 42
= 2.3853 ¥ 10-27 m 99. I.P. = –E1 = –13.6 ¥ 2 = - (for I.P., n
always = 1) n 12
e2 E = 217.6 eV
89. Total energy = - = - 3.4 eV = 21
2rn n hc
100. Required energy = 10–17 J = n ¥
-13.6 l
\ n2 = =4 6.625 ¥ 10-34 ¥ 3 ¥ 108
- 3.4 fi n¥ J = 10-17 J
n=2 550 ¥ 10-9
\ n = 28
The velocity in second orbit hc 1242 1242
101. E (eV) = = = = 1.77 eV
U1 2.18 ¥ 108 l l (nm) 700 nm
= = (cm sec -1 )
2 2 102. E = hn
= (6.62 ¥ 10–34 Js) (880 s–1)
h 6.626 ¥ 10-27 ¥ 2 = 5.831 ¥ 10–31 J.
l= =
mu 9.1 ¥ 10-28 ¥ 2.18 ¥ 108 1
Number of photons emitted = ¥ 10+31 = 1.71 ¥
= 6.6 ¥ 10–10 = 6.6 Å 30 5.831
10 per second.
90. Shape of s-orbital is spherical, so it is non–directional e 1
orbital. 103. For a proton (11 H), =
m 1
1 Ê 1 1ˆ e 2 1
91. = RH Z 2 Á 2 - ˜ 4
For an a-particle ( 2 He), = =
l Ë1 •¯ m 4 2
hc RH hcZ 2 (e / m) p 1/1
E= = = = 2 :1
l 1 (e / m)a 1/2

IIT JEE PC-V1_02.indd 35 3/21/2017 5:15:44 PM


Join our telegram channel: @Jee_neet_topper

2.36 Physical Chemistry-I for JEE (Main & Advanced)

h h h
104. The de Broglie equation is l = = l=
p mv mv
1 h
Here h and v are constant. So l µ . l= ; l =1Å
m 2eVm
Since the a-particle has the highest mass among the
150
given entities, it has the smallest de Broglie wave- On solving l = Å for electron
lengths. V
13.6 150
105. En = - 2 Z 2 , where n = 1 for ground state, 2 for first l=1Å= \ V = 150 Volt
n V
excited state, and so on. Z = 1 for hydrogen. 113. Number of unpaired electrons in Cr3+ = 3
1 Ê 1 1ˆ 114. Violates Hund’s law
106. = R• Á 2 - 2 ˜ for H atom.
l Ë n1 n2 ¯ 115. 1.73 = n(n + 2) V = 3d3 4S2
For the Balmer series, n=1
n1 = 2 and n2 = 5 (for the third line). Oxidation state = 4
1 Ê 1 1 ˆ 21 h
= R• Á 2 - 2 ˜ = R• 116. Dp ◊ Dx ≥
l Ë2 5 ¯ 100 4p
100 Given Dp = Dx
l=
21R• h
(mDv) 2 ≥
107. l = 3 mean f subshell have 7 orbital. 4p
Maximum multiplicity for the f orbitals (7 electrons) 1 h
= 2s + 1 = 2 ¥ 7/2 + 1 = 8 Dv ≥
2m p
Minimum multiplicity for the f orbital (1 electron) 117. For the second line of Lyman series of H,
= 2s + 1 = 2 ¥ 1/2 + 1 = 2 n1 = 1, n2 = 3
Z2 1 È1 1 ˘ 8R
108. En = -13.6 ¥ eV = R ¥ 12 Í 2 - 2 ˙ =
n2 lH Î1 3 ˚ 9
For the 3rd excited Bohr’s orbit of H atom 9
fi Z = 1, n = 4 \ lH = …(i)
8R
E1 = –13.6 eV
For 3rd excited state of hydrogen For the sixth line of Paschen series,
n1 = 3, n2 = •
12
E2 = -13.6 ¥ = - 0.85 eV 1 È1 1 ˘ È 1 1 ˘ RZ
2
42 = RZ 2 Í 2 - 2 ˙ = RZ 2 Í - ˙ =
lx Î3 • ˚ Î9 0 ˚ 9
-13.58
109. En = 9
n2 \ lx = …(ii)
RZ 2
at n = 1, E1 = –13.58 eV
(In primitive stage) n = 2, E2 = –3.395 eV But, lH = lX
Energy absorbed = –3.395+13.58 = 10.19 eV 9 9
i.e.; = \ Z2 = 8
n2 8R RZ 2
110. rn = r1 ¥
Z Z = 83
42 i.e., Li+2 – ion.
r2 = r1 ¥ = 0.106 nm
2 118. According to the question
n2 h
111. rn = r0 ¥ , here Z = 1 l=
Z p
rn µ n2 h
lA = …(i)
So r1 : r2 : r3 … = n12 : n22 : n32 … pA
= 12 : 22 : 32 h
= 1:4:9 lB = …(ii)
pB
112. If electron is accelerated by V volt
1 l A pB
K.E. = eV = mv 2 = l B = 10-7 m
2 lB pA

IIT JEE PC-V1_02.indd 36 3/21/2017 5:15:45 PM


Join our telegram channel: @Jee_neet_topper

Atomic Structure 2.37

119. We know that  1 fm = 10–15 m ª 5 fm


rn = r1 ¥ n2 134. (b) d = 20 nm
9r1 = r1 ¥ n2 20
\ n2 = 9 r= = 10 nm =100 A°
2
\ n=3 n2
 r = 0.529 ¥ A° For H atom Z = 1
13.6 Z
\ E3 = - 2 = -1.51 eV
3 100 = 0.529 ¥ n2 n = 14
120. According to Bohr’s atomic model Z2
mvr = nh/2p 135. (b) En = –13.6 ¥
n2
6.62 ¥ 10-34 1
4.2176 ¥ 10–34 = n2 ¥ E1(H) = – 13.6 ¥ = –13.6 eV
2 ¥ 3.14 1
\ n2 = 4 4
1 È1 1˘ E2(He+) = – 13.6 ¥ = –13.6 eV
= R ¥ 12 Í 2 - 2 ˙ 4
l Î3 4 ˚ 32
E3(Li2+) = – 13.6 ¥ = 2 –13.6 eV
\ l = 18.75 ¥ 10–7 m 3
hc 42
121. E3 – E1 = DE = E4(Be3+) = – 13.6 ¥ = –13.6 eV
l 42
hc 1242 1242 \ B is the answer.
l= or l (nm) = =
DE DE (rev) 12.4 ev 136. (c) E = –78.4 kcal/mol
l = 1030 Å = 103 nm Z2
En = –313.6 ¥ kcal/mol
122. hn = work function + K.E.; n2
Given K.E. = 0; 1
for H atom Z = 1 –78.4 = 313.6 ¥
1242 n2
Thus, hn = 4 eV or 4 = 313.6
l (nm) n2 = n=2
78.4
1 Ze 2 Ze 2 137. (a) Vn = 2.188 ¥ 106 ¥ m/sec.
123. K.E. = mu 2 = , T.E. = - \ (A)
2 2r 2r
V3 (Li 2+ ) Z 3 / n3 3/3
124. The state corresponds to the 3d atomic orbitals. = = V(Li2+) = V
125. The 21st electron corresponds to 3d1. For this orbital, V1 (H) Z1 / n1 1/1
1 138. (a) Let state (A) = n1
n = 3, l = 2, m = 2, s = + . State (B) = n2
2
126. Mn4+ (21 electrons) = [Ar]3d3 contains three unpaired r1 – r2 = 624 r0
electrons. n12 0.529n22 0.529 ¥ 1
127. The spin-only formula is given by mspin only 0.529 ¥ - = 624 ¥
Z Z Z
=n(n + 2) BM n12 - n22 = 624
n = No of unpaired e– n1 = 25
The value of m = 2.83 BM corresponds to the presence n2 = 1
of two unpaired electrons. So the ion is Ni2+ (3d8). 25 Æ 1
128. For the 3p orbital, n = 3, l = 1. 139. (c)
The number of spherical nodes = n – l – 1 = 3 – 1 – 1 = 1 (a) Energy of ground state
129. Spin multiplicity = (2S + 1). He+ = –13.6 ¥ 4 eV = –54.4 eV
rA 0.529 ¥ 1
130. (a) = 105 (b) P.E. of first orbit of H-atom = 2T.E. =
rN Z
–2 ¥ 13.6 eV = –27.2 eV
3
VA Ê rA ˆ (c) Energy of second excited state
= = (105 )3 = 10-15
VN ÁË rN ˜¯ Z2 (2) 2
= -13.6 ¥ = - 13.6 ¥
131. (a) neon and O– 2 have 10 electron n2 (3) 2
C 3×108 4
132. (a) l = = = 0.75 m = -13.6 ¥ = - 6.03 eV
n 400×106 9
133. (a) R = R0 A1/3 = 1.33 ¥ 10–13 ¥ (64)1/3 cm (d) I.E.= –E1 = 21.8 ¥ 10–19 ¥ 4J = 8.7 ¥ 10–18 J
= 5.32 ¥ 10–13 ª 5 fm

IIT JEE PC-V1_02.indd 37 3/21/2017 5:15:47 PM


Join our telegram channel: @Jee_neet_topper

2.38 Physical Chemistry-I for JEE (Main & Advanced)

140. (d) For dyz, xy and xz are nodal palne


node = (n – l – 1) = 6 – 2 – 1fi 3 LEVEL II
141. (b) x h y + 42 He
1. (d) last e– enter in 2P subshell
y h 8O18 + 1H1
\ l=1
y = 9y19 2. (d)
x = 11x23 Pd 1s2 2s2 2p6 3s2 3p6 4s2 3d10 4p6 5s0 4d10
Hence x = Na (I) l = 0
Na present in third period 2 2 2 2 0 = 8 electrons
Number of neutron = 23 – 11 fi 12
(II) Z is different, therefore, Zeff will be different.
4.6 (III) Configuration of 26Fe Æ [Ar] 4s2 3d5
Mole of Na = fi 0.2
23 Configuration of Ni+2 Æ [Ar] 3d8 4s0
Mole of neutron fi 0.2 ¥ 12 fi 2.4 Number of unpaired electrons are different.
hc 1240 1240 (IV) 57 electron (La ) 5d1
142. (c) E = fi eV E = fi 40 eV
l lnm 31 n=5
40 = 12.8 + K.E. Êlˆ
l=2 ÁË ˜¯ = 0.4
K.E. = 40 – 12.8 = 27.2 eV n
+2
K.E. = 27.2 ¥ 1.6 ¥ 10–19 3. (a) A have 22 electron
1 A has 24 electron / 24 p+
27.2 ¥ 1.6 ¥ 10–19 = ¥ 9.1 ¥ 10–31 ¥ v2 n0 = 26
2
(i) Mass number of A = 50
v = 2.18 ¥ 2 ¥ 106 m/s Atomic number is 24
1 v z /n 3d54s1
143. (d) Frequency = µ µ 2 +2
T r n/z (iv) A 1 electron is removed from s and 1 from d.
4. (a) Ga (31) 1s2 2s2 2p6 3s2 3p6 4s2 3d10 4p1
1 z2 n3 1/4 1
Frequency = µ 3 Tµ 2 = fi n=4
T n z 8/1 32 l=1 n+l+m=6
144. (a) s-orbital is spherical hence it is non-directional. m=1
h 5. (b)
145. (c) Dx = 2Dp Dx ◊ 2Dp =
4p (a) Fe+3 [Ar]3d5
h h Co+3 [Ar] 3d6
2 (Dp)2 = (Dv)2 = More the number of unpaired electron, more will be
4p 8p m 2
multiplicity.
1 h 1
Dv = Dv = h Ti +3[Ar]3d1 ¸
2m 2p 2m ÔÔ
(b) Cr + [Ar]3d 5 ˝ are paramagnetic
Ô
Sc +2 [Ar]ed1 Ô˛

45° (c) Mg [Ne] 3s2


146. (c)
n = 3, l = 0, m = 0
\ n+l+m=3
(d) Cl (1s2) (2s2 2p6) (3s2 3p5)
s = 6 ¥ 0.35 + 8 ¥ 0.85 + 2 ¥ 1 = 10.9
147. (c) n = 5 Zeff = 17 – 10.9 = 7.1
l = 0, 1, 2, 3, 4, 6. (a) Element is Na
s, p, d, f, g \ number of neutron = 12
148. (Theory Based) \ number of proton and electron = 11
149. (c) From (n + l) rule (a) Mg24 also has 12 neutrons
150. (d) The value of l = 0 to (n – 1) (b) 3s1 is last electron
Number of electron for given value of l = 2 (2l + 1) (c) H2O has 10 electrons
l = ( n –1) (d) Since atoms have 1 unpaired electron \ it is para-
Hence  2(2l + 1) magnetic.
l=0

IIT JEE PC-V1_02.indd 38 3/21/2017 5:15:49 PM


Join our telegram channel: @Jee_neet_topper

Atomic Structure 2.39

7. (b) 4 - a0 / a0
(a) 12Mg 1s2 2s2 2p6 3s2 (Diamagnetic) 4p a02 ¥ e
p1 a03
(b) 20Ca 1s2 2s2 2p6 3s2 3p6 3d2 (Paramagnetic) =
p2 a02 4
(c) 28Ni 1s2 2s2 2p6 3s2 3p6 3d10 (Diamagnetic) 4p ¥ 3 e -2 a0 /2 a0
(d) 30Zn 1s2 2s2 2p6 3s2 3p6 3d10 4s2 (Diamagnetic) 4 a0
8. (a) For 2.84 B.M., 2 unpaired electrons are required. p1 e -2 4
= =
Fe2+ [Ar] 3d5 p2 1
e -1 e
Cr [Ar] 3d5·4s1 4
Cr+3 [Ar] 3d3 15. (a) See graph in theory
Ti +2
[Ar] 3d2 16. (b) Dx = 0.0001 Å
Mn +2
[Ar] 3d5 DP = ?
+3 h h
V [Ar] 3d2 Dx ¥ Dp ≥ ; DP =
9. (a) S = 1s22s22p63s2 3p2 4p 4p ¥ 0.0001 Å
a0 = 0.529 Å
h
1 1 1 1 1 1 1 1 1 P=
2p ¥ 0.529 Å
S = +1/2
minimum (7) e– h
S = +1/2 [16 – 7] = 9 DP 4p ¥ 0.0001 Å
=
Maximum (9e–) P h
(b) e with m = 0 present in s-orbital is not compulsory 2p ¥ 0.529 Å
as the value of n increases m also be 0. DP 2 ¥ 0.529
= = 2645
(c) Na+1 Mg+2 all isoelectronic species have same P 4 ¥ 0.0001
s but different Zeff
h
(d) Number of subshell nth shell is equal to n. 17. (c) l =
10. (d) l = 2 2m ¥ e ¥ V
Fe (26) = 1s2 2s2 2p6 3s2 3p6 3d6 4s2 h
l=
Fe2+ (24) = 1s2 2s2 2p6 3s2 3p6 3d6 V
2 ¥ m¢ ¥ 2e ¥
l = 2 fi d 8
Electron present in (l = 2) d-orbitals = 6 V
11. (a) 1s4 2s4 2p8 3s4 3p8 4s2 m¢ ¥ 2e ¥ = m ¥ e ¥V
8
v m¢ = 4 m
12. (d) Hint Use
2p r 1242
13. (b) DE +2 = DEH 18. (a) Maximum KE of Ejected e– = – 4.65
Li 253.7
= 0.245 ev
È1 1˘ È1 1˘
13.6 ¥ 32 Í 2 - 2 ˙ = 13.6 ¥ 12 Í 2 - 2 ˙ 19. (c) Spherical node : Y2 = 0
Î n1 n2 ˚ Î1 4 ˚ s2 – 4s + 3 = 0; s2 – 3s – s + 3 = 0
È 1 ˘ È1 1˘ s (s – 3) – 1 (s – 3) = 0; (s – 1) (s – 3) = 0
Í 2 2˙
=Í 2 - 2˙ 2 Zr
Í Ê n1 ˆ - Ê n2 ˆ ˙ Î1 4 ˚ s = 1, 3; s= = 1, 3
a0
ÍÎ ÁË 3 ˜¯ ÁË 3 ˜¯ ˙
˚ 3 a0 1 a0
r= ,
fi n1 = 3 2 Z 2 Z
n2 = 12 nh
14. (c) Probability of finding and electron at a particular 20. (b) Let the orbits are n and (n + 1) mn1r1 = …(i)
2p
distance = 4pr2R2 (n + 1)h
4 mn 2 r2 = …(ii)
P1 = 4pr2R2 = 4pr2 ¥ 3 e -2 r / a0 2p
a0 The difference in angular momentum will be (ii) – (i)
4 -2 a0 / a0 (n + 1)h nh Ê h ˆ
at r = a0 p1 = 4p a02 ¥ e or (mv2 r2 - mv1r1 ) = - =Á ˜
a03 2p 2p Ë 2p ¯

a02 4
-2 a
0 1 Ê 1 1ˆ
at r = a0/2 p2 = 4p ¥ 3 e 2 a0 21. (a) = R ¥ (1) 2 ¥ Á 2 ¥ ˜ ; lmin = 911.7 Å
4 a0 lmin Ë1 •¯

IIT JEE PC-V1_02.indd 39 3/21/2017 5:15:51 PM


Join our telegram channel: @Jee_neet_topper

2.40 Physical Chemistry-I for JEE (Main & Advanced)

35. (b) Ti = 1s2 2s2 2p6 3s2 3p6 3d2


2.19 ¥ 106 ¥ Zm/sec.
22. (a) n n =
n
For n = 2 of hydrogen atom n=2
6
2.19 ¥ 10 ¥ m/sec Magnetic moment = n(n + 2) fi 2(4) = 8 = 2.83 BM
n2 = = 1.09 ¥ 106 m/sec.
2 n3 T1 n13
23. (d) n1 + n2 = 4 36. (d) T µ = = 1/8
z2 T2 n23
n2 - n1 = 2
37. (a) E• – E1 = hn1, fi E1 fi hn1
2n2 = 6 E2 – E1 = hn2
\ n2 = 3 E• – E2 = hn3, fi E2 fi hn3
For the third line of the same series; –hn3 + hn1 = hn2
n1 = 1 and n2 = 4
v2 = v1 – v3
4
v3 = v1 – v2
3
2 hc
3rd Line 38. (d) EC – EB = ...(i)
1 l1
1 hc
È1 1˘ È1 1˘ EB – EA = ...(ii)
= RZ 2 Í 2 - 2 ˙ = R ¥ 9 Í 2 - 2 ˙ l2
l n
Î 1 n2˚ Î1 4 ˚
hc
l = 1080 nm EC – EA = ...(iii)
l3
h
24. (d) l = v l = add equations (i) and (ii)
mv
Ê1 1ˆ
EC – EA = hc Á + ˜
h Ë l1 l2 ¯
l 2 = fi l =
m put in equation (iii)
25. (b) According to Schrodinger’s model e– behave as Ê1 1 ˆ hc
hc Á + ˜ =
wave only. l
Ë 1 l 2¯ l3
26. (b) The maximum probability of finding an electron is
described by the orbital, which is denoted by Y2. 1 l1 + l2 ll
= fi l3 = 1 2
h l3 l1l2 l1 + l2
27. (a) lm = le l=
mv hc
h h ve mn 39. (d) DE =
= = l
mc vc mn vn vn mc hc
DE = (for H atom)
28. (b) (Y) is the solution of Schrodinger wave equation. l1
29. (d) 2pr = nl [as per de Broglie theory] hc
DE ¥ z2 = (for He+ ion)
30. (b) my = 0.25 mx, vy = 0.75 vx l2
h h h hc hc l
l= lx = , ly = ¥4= fi l2 = 1
mv m v
x x m yvy l1 l2 4
h 40. (b) First excitation potential
ly = ly = 5.33 A
0.25M x ¥ 0.75vx = E2 – E1 fi – 4 + 16 fi 12 eV
41. (d) n2= 4, n1 =3; n=•
31. (a) Orbital angular momentum = l (l + 1) 
s p d f n2 = 5 n1 = 4; E/4
l=0 1 2 3 n2 = 6 n1 = 5; E
n Æ (n – 1) (n ≥ 4) 3E/4
m –1
32. (c) m = (2l +1) fi l = 42. (c) n2 = 5, n1 = 1
2 Dn( Dn + 1)
33. (d) Mn4+ = 1s2 2s2 2p6 3s2 3p6 3d3 Total number of spectrum line are
OR S (5 – 1) fi S4 2
OR S4 fi 4 +3 + 2 + 1
lyman Balmer Paschen brackett
34. (a) According to (n + l) rule, after np, (n + l) s always
Three line in visible reigon.
filled.

IIT JEE PC-V1_02.indd 40 3/21/2017 5:15:54 PM


Join our telegram channel: @Jee_neet_topper

Atomic Structure 2.41

LEVEL III 1 h
Dp =
2¥2 p

(Match the Column) 1 h


Dv =
2 2m p
1. [(A) P (B) P, Q, R, S, T (C) Q, R, S (D) P ]
2. A – r.t B-p C-q, s D – q 20. (b) Dv = 0.3
3. A – p B- p C-p, q, r, s D – p h 6.6 ¥ 10-27
4. A – r, p B – r, t C – s D – q Conceptual Dx = = = 1.92 cm
4p mDv 4 ¥ 3.14 ¥ 9.1 ¥ 10-28 ¥ 0.3
5. A – r B - q C – p, t D - s
(A) for H , radius of second orbit 21. (d) When Dx = 0, Dp = •
22 Passage 5
= 0.53 ¥ = 0.53 ¥ 4 Å
1 h
22. (d) Orbital angular momentum (O.A.M) = l (l + 1)
6. (A) Æ t; (B) Æ s ; (C) Æ u ; (D) Æ q ; (E) Æ p; (F) Æ r 2p
7. (A) Æ u ; (B) Æ s ; (C) Æ p ; (D) Æ t ; (E) Æ q ; (F) Æ r n = 5, So l = 1,2,3,4. For maximum
8. (A) Æ q ; (B) Æ p ; (C) Æ q, r ; (D) Æ r, s h h
9. (A) Æ r ; (B) Æ s; (C) Æ p; (D) Æ q O.A.M = 4(5) = 20
2p 2p
10. A Æ P, BÆp, q, s, CÆp, r, D Æ q, s
Radial node = (n – l – 1) h
23. (b) m = l (l + 1) cos q ¥
Angular node = l 2p
4s, 5px, 6dxy having 3 radial node. l = 1 for p-orbital
angular node in all ‘s’ orbital in zero. h
m = 1(2) cos 45∞ ¥
Passage-1 2p
h
11. (b) 1st atom 2nd 3rd atom Z - component of O.A.M =
2p
24. (a) 8 = n(n + 2); n = 2
Number of unpaired electrons = 2
1 1
12. (b) s= + =1
2 2
150 h h
13. (c) l = Ls = s ( s + 1) = 2
V 2p 2p
V = 24 volt \ energy of free electron = 24 eV, 25. (a) Lyman series
È1 1˘ 1 È1 1˘
DE = 13.6 ¥ (Z)2 Í 2 - ˙ = n1 = RH ◊ 12 Í 2 - 2 ˙
Î6 •˚ l1 Î1 n2 ˚
= 1.51 eV
Total energy released = 24 – (–1.51) = 25.51 Balmer series
14. (b) (n – 2) 1 È1 1˘
6–2=4 = n 2 = RH ◊ 12 Í 2 - 2 ˙
l2 Î2 n2 ˚
15. (a) Minimum energy transition for 6 Æ 5
For highest energy transition
Passage 3 For Lyman series n2 = •
16. (c) Maximum number of electrons in an orbit is = 2n2 = 2 For Balmer series n2 = •
¥ 32 = 18
17. (d) Maximum number of electrons in a subshell = 2[2l n1 4
\ =
+ 1] = 4l + 2 = 4 ¥ 3 + 2 = 14 n2 1
18. (d) For 2p orbital n = 2, l = 1
1 È1 1˘
Passage 4 26. (b) = n = RH Z 2 Í 2 - 2 ˙
l
19. (d) Dx = 2Dp ÎÍ n1 n2 ˙˚
h For lowest energy transition of ion in Lyman series
Dx ¥ Dp =
4p È1 1˘
n = RH 22 Í 2 - 2 ˙ = 3RH
h Î1 2 ˚
2( Dp ) 2 =
4p

IIT JEE PC-V1_02.indd 41 3/21/2017 5:15:55 PM


Join our telegram channel: @Jee_neet_topper

2.42 Physical Chemistry-I for JEE (Main & Advanced)

Passage 7 39. (c) mass number is the sum of n + p


27. (c) Cr3+ = 3d3 (Valance shell). 1242
n = 3 (Unpaired electrons) 40. (a) nm = 365.3 nm
3.4
m s = 15 BM 41. (a) Hund’s Rule
42. (b) (n + l) Rule and Hund’s Rule
43. (a) Spherical nodes = n – l – 1
28. (b) 2 –1 – 1 = 0
for p orbital l = 1
\ Spherical node for p orbital = n – 2
Fe3+ = 3d5 (Valance shell) 44. (b) Ruther ford’s experiment
45. (e) concept
1
s= in strong Oh magnetic field 46. (b) Spherical nodes = n – l – 1
2 angular node = l
Spin multiplicity = 2(s) +1 = 2 47. (a, b, d)
29. (a) Momentum = 2mK .E. Z Z 1
(a) v = 2.18 ¥ 106 ¥ fi v µ or v µ
Pe = Pa n n n
2me ◊ K.E.e = 2ma ◊ K.E.a v v Z /n Z2
(b) f = or f = µ 2 f µ 3
meK.E.e = maK.E.a 2p r r n /Z n
K.E.e ma È n3 ˘
= (c) r µ n 2 / Z ÍT µ 2 ˙
K.E.a me Î Z ˚
Mass of a particle is more than mass of electron. mn 2 n 2 (Z 2 / n2 ) Z3
m K.E.e (d) F = \F µ µ 2 Fµ 4
Thus, a > 1; thus >1 r r n /Z n
me K.E.a
So ans (A, B, D)
Thus statement 1 is true.
48. (a, c, d) B.E. = I.E.
Statement 2 is also true.
(I.E.)any atom = (I.E.)H ¥ z2
30. (d) Aufbau principle
31. (c) Pairing is favourable when all orbitals are half 122.4
= z2
filled. 13.6
32. (c) The Balmer series is born of transition of electron z2 = 9 z = 3
into the second orbit from higher orbits. E2 – E1= 122.4 – 30.6 = 91.8 eV
33. (b) Both statements are independently true. 49. [a, b, c, d] 10 cm–1 = 1000 m–1
34. (c) Cu+ = 1s2 2s2 2p6 3s2 3p6 3d10 hc
\ E = hn = = hc
í
All the electrons are paried; hence it is paramagnatic. l
(b) For energy of second orbit = 6.66 ¥ 10–34 (Js) ¥ 3 ¥ 108 (ms–1) ¥ 1000 (m–1)
22 = 2 ¥ 10–22 J per photon
= -13.6 ¥ 2 = -13.6 eV
2 = 2 ¥ 10–15 ergs per photon
42 = 2 ¥ 10–22 ¥ 6.02 ¥ 1023 J mol–1
(c) For radius of fourth orbit = 0.53 ¥ = 0.53 ¥ 4 Å
4 = 1.2 ¥ 10–1 kJ mol–1
Also, energy of fourth orbit
= 2.9 ¥ 10–2 kcal mol–1
42 50. [b, c, d] According to emission photoelectric effect
= -13.6 ¥= -13.6 eV 1
42 hn = hn0 + mV2
(d) For, velocity of electron in third orbit = 2
hc
3 E of incident radiation =
2.18 ¥ 108 ¥ = 2.18 ¥ 108 Å l
3 6.626 ¥ 10 –34 ¥ 3 ¥ 108
Assertion and Reason =
200 ¥ 10 –10
Nodal plane in yz plane
35. (a) Px = 9.94 ¥ 10-18 J
5 ¥ 1.6 ¥ 10 –19
36. (a) Pauli rule n0 = = 1.2 ¥ 1015 sec–1
37. (b) Lobes = 2l 6.626 ¥ 10 –34
38. (a) Conceptual 1 2
mv = 9.94 ¥ 10–18 – 8 ¥ 10–19 = 9.14 ¥ 10–18
2

IIT JEE PC-V1_02.indd 42 3/21/2017 5:15:57 PM


Join our telegram channel: @Jee_neet_topper

Atomic Structure 2.43

69. (6) Hint: n = 4, l = 1 represents 4P – subshell contain-


2 ¥ 9.14 ¥ 10 –18 ing six electrons. Thus, three will be six elements hav-
v= = 4.47 ¥ 106 m/sec
9.1 ¥ 10 –31 ing 4P1 to 4P6 electronic configuration.
51. (A, D) Both Fe3+ and Mn2+ have 5 unpair electrons. h
70. (0) Hint: ml = l (l + 1) (l = 0 for s -orbital)
Cu2Cl2 is colourless because Cu+ has no unpair elec- 2p
trons. 71. (5) Hint: Cu+ = [Ar]3d104s0
MnO -4 has no unpair electrons in Mn+7, due to charge 1
Five d – electrons have + spin
transfer spectra it is coloured. 2
m = n(n + 2) BM and n = 4 1
Five d – electrons have – spin
2
for Fe+2 and CO+3
72. (12) Hint: n = 4 l = 0 me = 0
52. (4) If m = + 3 (maximum), then l = 3 and n = 4
l = 1 me = –1, 0, +1
Number of waves in an orbit = n
l = 2 me = –2, –1, 0, +1, +2
53. (6) Number of spherical lines produced
l = 3 me = –3, –2, –1, 0, +1, +2, +3
(n – n )(n – n + 1) (5 – 2)(5 – 2 + 1)
= 2 1 2 1 = =6 There are three orbitals having me = +1, and three orbit-
2 2 als having me = –1,
54. (0) Orbital angular momentum Thus maximum number of electrons in them will be 12.
h h 73. (1) Hint: Number of radial nodes
= l (l + 1) = 0(0 + 1) =0
2p 2p = n – l – 1 = 3 – 1 – 1 (for 3P, n = 3, l = 1).
h 74. Distance to be travelled from Mars to Earth
55. (2) mvr = n = 8 ¥ 107 km
2p
h h = 8 ¥ 1010 m
=n ,n=2  velocity = 3 ¥ 108 m/sec
p 2p
8 ¥ 1010
56. (0) Number of radial modes = n – l – 1 \ Time = Distance/Velocity =
2–l–1=0 3 ¥ 108
-6.39 ¥ 10-19 = 2.66 ¥ 102 sec.
57. (4) Number of electron = =4
-1.60 ¥ 10-19 75. (a) I.P. = DE = E• – E1 = 0 –(–15.6) = 15.6 l.v.
58. (3) Number of waves = n 1= •

60. (1) Number of radial nodes = n – l – 1 = 2 – 0 – 1 = 1 (b) n = • n = 2


DE = [0 – (–5.3)] = 5.3 l.v.
61. (4) n(n + 2) = 24
1240 1240
n(n + 2) = 24, n = 4 DE = l= = 233.9 nm
l (nm) 5.3
62. (2) y420 is 4d orbital l = 2
63. (2) n – l – 1 = 3 – 0 – 1 = 2 (c) |DE3 Æ 1| = |–3.08 – (–15.6)|
64. (5) l = 2 = 15.6 – 3.08 = 12.52 e.v.
Number of d-electrons = 5 1240 12.52 1
= = = (n.m)
65. (3) En = –13.6 + 12.09 = –1.51 l 1240 l
E -13.6 l = 1.808 ¥ 107 m–1
En = 21 \ n 2 = = 9, n = 3
n -1.51 (d) (i) E = –15.6 – (–6) = –15.6 + 6 = –9.6
66. (0) Hint: m = n(n + 2) = 35 BM (ii) E = –15.6 – (–11) = –15.6 + 11 = –4.6
n=5 n hc
76. E =
i.e., 5 unpaired ee are present in (n – 1)d l
shell of M+3 transition metal cation n ¥ 6.625 ¥ 10-34 ¥ 3 ¥ 108
so it is Fe+3 10–17 =
6 2 550 ¥ 10-9
26Fe Æ [Ar]3d 4s
5 0 n = 27.67 \ 28
Fe Æ [Ar]3d 4s
Circumference 77. 330 J = n(hn)
67. (2) Hint: Number of waves n = 330 J = n [6.62 ¥ 10–34 ¥ 5 ¥ 1013]
Wavelength
400 330
n= = 2 (or) nl = 2p r = n = 1022
200 6.62 ¥ 10-34 ¥ 5 ¥ 1013
n ¥ 200 = 400 nhC 3.15 ¥ 10 –14 ¥ 850 ¥ 10 –9
n=2 78. E = n=
68. (4) Hint: m = 3, l = 3, n=4
l 6.62 ¥ 10 –34 ¥ 3 ¥ 108
For n = 4 number of waves will be 4. n = 134.8 ¥ 103 n = 1.35 ¥ 105

IIT JEE PC-V1_02.indd 43 3/21/2017 5:16:00 PM


Join our telegram channel: @Jee_neet_topper

2.44 Physical Chemistry-I for JEE (Main & Advanced)

1 È1 1˘ = 5.425 ¥ 10–12 ergs


79. = Rh Z 2 Í 2 - 2 ˙
l Î n1 n2 ˚ 6.625 ¥ 10 –34 ¥ 3 ¥ 108
(b) l =
n1 = 3 for paschan 5.425 ¥ 10-19
80. n2 = 3 n1 = 2 [first line] = 3.66 ¥ 10–7 meter = 3.7 ¥ 10–5 cm
n2 = 4 n1 = 2 [second line] È 1 1˘
1 86. DE = I.E. Í – ˙
È1 1˘ 2 Æ1 Î 4 1˚
= RH Í – ˙
l Î4 9˚ È 1 1˘ hc
1 2.17 ¥ 10–11 erg/atom Í – ˙ =
È1 1˘ Î 4 1 ˚ l ( m)
Å = RH Í – ˙ (i)
6565 Î4 9˚ –34 8
È 1 1˘ 6.626 ¥ 10 ¥ 3 ¥ 10
1 È1 1 ˘ 2.17 ¥ 10–11 ¥ 10–7 J Í – ˙ =
= RH (ii) Î 4 1˚ l
l ÍÎ 4 – 16 ˙˚
6.626 ¥ 1034 ¥ 3 ¥ 108 ¥ 4 6.626 ¥ 4 ¥ 108
5 l= =
2.17 ¥ 10 –18 ¥ 3 2.17
l 36 5 ¥ 16
= = l = 4863 Å = 12.20 ¥ 10–8 m = 1220 Å
6565 3 36 ¥ 3
16 Z 2.18 ¥ 106
81. 3 Æ 2 87. Vn = 2.18 ¥ 106 ¥ =
n n
1 È1 1˘ È1 1˘
= RH ¥ Z 2 Í 2 – 2 ˙ = RH ¥ Í – ˙ ...(i) 2.18 ¥ 10 6
1 v
l1 Î n1 n 2 ˚ Î4 9˚ = =
n 275 3 ¥ 108
1 È1 1 ˘
2 Æ1 = RH ¥ 4 Í – ˙ ...(ii) 2.18 ¥ 106 3 ¥ 108
l2 Î1 4 ˚ =
n 275
(l1 – l2) = 133.7 nm ...(iii) n = 1.99  2
We solve the three equations and we get
88. Z = 3, n1 = 1, n2 = 3
R = 1.096 ¥ 107 m–1
È1 1˘ È1 1 ˘
È1 1 ˘ En = 13.6 ¥ (Z2) Í 2 – 2 ˙ = 13.6 ¥ 9 Í – ˙
82. DE = 13.6 Í – ˙ ¥ 96.3368 kJ/mole n
Î 1 n2˚ Î1 9 ˚
Î9 4 ˚
8
È4 – 9˘ = 13.6 ¥ 9 ¥ = 108.8 eV
= 13.6 Í ¥ 96.368 = 182.074 9
Î 36 ˙˚
È1 1˘ È1 1 ˘
= 1.827 ¥ 105 J/mole 89. (i) En2 Æ n1 = 13.6 ¥ Z 2 Í 2 – 2 ˙ = 13.6[1]2 Í – ˙
Î n1 n2 ˚ Î1 4 ˚
hc
83. IE per mole = N A ¥ 3
l = 13.6 ¥ 1 ¥ = 10.2 eV
4
6.022 ¥ 1023 ¥ 6.625 ¥ 10-34 ¥ 3 ¥ 108
= J 1 È1 1˘
854 ¥ 10-10 (ii) = RH Z 2 Í 2 – 2 ˙
l Î n1 n2 ˚
= 0.14 ¥ 107 = 14 ¥ 105 J = 1400 KJ/Mole
1 7 2 3
84. Radius = 16 ¥ radius of H in ground state = 16 ¥ 0.529 –8 = 1.09 ¥ 10 ¥ Z ¥
3 ¥ 10 4
n2
16 ¥ 0529 = 0.0529 ¥ \ Z=2
Z 90. 1.8 mole = (1.8 NA) atoms
n2 27% = IIIrd energy level = 1.8 ¥ NA ¥ 0.27 = 0.486 NA
16 = n=4
1 15% = IInd energy level = 1.8 ¥ NA ¥ 0.15 = 0.27 NA
Z2
T.E. = –13.6 ¥ 2 e.v. = –0.85 e.v. DE = DE1 + DE2
n 3Æ1 2 Æ1
–12
– 21.7 ¥ 10 È 1 1˘
85. En = 1 erg = 10–7 Joule = 1.8 ¥ NA ¥ 0.27 ¥ IE + Í – ˙
n2 Î 9 1˚
– 21.7 ¥ 10 –12 È 1 1˘
En = + 1.8 ¥ NA ¥ 0.15 ¥ IE Í – ˙
4
Î 4 1˚
È – 21.7 ¥ 10 –12 ˘ 21.7 ¥ 10 –12 = (0.432 NA + 0.20 NA) ¥ IE = 56.65 ¥ 1011 erg.
J.E. = 0 - Í ˙=
Î 4 ˚ 4

IIT JEE PC-V1_02.indd 44 3/21/2017 5:16:02 PM


Join our telegram channel: @Jee_neet_topper

Atomic Structure 2.45

91. Number of atom in third orbit = 0.5 NA The energy of photon which can cause ejection of elec-
Number of atom in second orbit = 0.25 NA tron
Total energy evolve = 0.5 NA(E3 – E1) + 0.25NA(E2 – E1) = 2.179 ¥ 1018 + 1.092 ¥ 10–18
Ê hˆ = 3.271¥ 10–18J
92. Angular momentum = n Á ˜
Ë 2p ¯ 97. Energy required per molecule in the following process
is given as
Ê hc ˆ (1) 2
ÁË ˜¯ = – 3.4 eV –3.4 = –13.6 ¥ hn
O 2 ææÆ O + O *
l n2
– 3.4 1 13.6 h ¥ c 6.626 ¥ 10 –34 ¥ 3 ¥ 108
= 2 2
n = =4 E= =
–13.6 n 3.4 l 174 ¥ 10 –9

n2 = 4 fi n = 2 = 11.424 ¥ 10–19 J/molecule


Energy for O Æ O* is 3.15 ¥ 10–19 Joule
Ê 6.626 ¥ 10 –34 ¥ 7 ˆ h 6.62 ¥ 10 –39 ¥ 7 Thus, energy O2 Æ 2O will be
= 2Á ˜ = or
Ë 2 ¥ 22 ¯ p 2 = 11.424 ¥ 10–19 – 3.1 ¥ 10–19
= 8.274 ¥ 10–19 J/molecule
1240 1 4.5
93. 4.5 eV = = = 0.0036 nm–1 = 8.274 ¥ 10–19 ¥ 6.023 ¥ 1023 ¥ 10–3 kJ/mole
l (nm) l 1240 = 498.3 kJ/mole
Dn( Dn + 1)
94. = 15 1 V0
2 98. V2 = V0 ¥ =
2 2
Dn (Dn + 1) = 30 = 5 ¥ 6 x=v¥t
\ n1 = 1 and n2 = 6
1 È1 1 ˘ V0 Ê V ¥ 10 –8 ˆ
= RH ¥ Í – ˙ x= ¥ 10 –8 sec = Á 0 ˜¯ m
lÅ Î1 36 ˚ 2 Ë 2
1 1 35 35 ¥ 2496 2pr distance covered in 1 round
= ¥ =
x 912 36 32832 V0 ¥ 10 –8 V ¥ 10 –8 1
\ distance covered in 0 ¥
l = 932 Å 2 2 2p r
r2 = r0 ¥ n2 = 4r0
95. The condition can be met by taking first derivative P(r)
w.r.t r and setting the same equal to zero, i.e., V0 /2 ¥ 10 –8 V0 ¥ 10 –8 ¥ 1
so, number of revolutions = =
v
dP(r ) 1 ÈÊ 1 ˆ 2 – r / a0 ˘ 2p ¥ 4r0 2 ¥ 2p ¥ 4r0
= ÍÁ 3 ˜ e ˙=0
dr dr ÍÎË p a0 ¯ ˚˙ 2.18 ¥ 106 ¥ 10 –18
=
2 ¥ 2 ¥ 3.14 ¥ 4 ¥ 0.529
\ r = a0 ( e -2 r / a0 π 0)
96. The energy of photon which can cause ejection of elec- 2.18 ¥ 10 –12
= = 0.838 ¥ 1019 = 8 ¥ 106
tron = kinetic energy of ejected electron + ionisation 2.6 ¥ 10 –21
energy …(i)
Ionisation energy = 13.6 eV V
99. n =
13.6 l
= J = 2.179 ¥ 10–18 J …(ii)
6.24 ¥ 1018 E of Ist Bohr orbit = –13.6
Here 6.24 ¥ 1018 eV = 1 J
h 6.626 ¥ 10 –34 ¥ 3 ¥ 108
de Broglie wavelength = l = -13.6 =
mv l
1240
h 6.626 ¥ 10 –34 or -13.6 =
\ v= = l (in nm)
ml 9.1 ¥ 10 –31 ¥ 4.7 ¥ 10 –10
= 0.15492 ¥ 107 m/sec 1240
l= ¥ 10
1 136
K.E. = mv 2
2 l = 91.17 (nm) = 912 Å
1
= ¥ 9.1 ¥ 10–31 ¥ (0.15492 ¥ 107)2 1
2 2.18 ¥ 106 ¥
n= 1 = 2.3 ¥ 1013 Hz
= 1.092 ¥ 10–18J …(iii)
Now from Eq. (i) 912 ¥ 10-10

IIT JEE PC-V1_02.indd 45 3/21/2017 5:16:04 PM


Join our telegram channel: @Jee_neet_topper

2.46 Physical Chemistry-I for JEE (Main & Advanced)

Ê 1 1ˆ hc
100. DE = 47.2 eV = 13.6 Z2 Á - ˜ 8. DE = E3 – E2 =
Ë 4 9¯ l
hc
47.2 5 2 = –2.41 ¥ 10–12 – (5.42 ¥ 10–12) =
= Z l
13.6 36
6.626 ¥ 10 –34 ¥ 3 ¥ 108
Z2 = 25 = 3.01 ¥ 10–12 ¥ 10–7 =
\ Z=5 l
–26 –7
101. DE = Z2 ¥ 10.2 eV = 4 ¥ 10.2 eV 6.626 ¥ 10 ¥ 3 6.626 ¥ 10 ¥ 3
l= =
3.01 ¥ 10 –19 3.01
1 –7
K.E. = mV 2 = 4 ¥ 10.2 - 13.6 eV = 6603 Å = 6.603 ¥ 10 m
2
È1 1˘
2 ¥ 27.2 ¥ 1.6 ¥ 10-19 9. DE = 2.17 ¥ 10–18 Í 2 – 2 ˙
\ V= -31
= 3.09 ¥ 106 m/sec. Î n1 n2 ˚
9.1 ¥ 10
hc È 1˘
= 2.17 ¥ 10 –18 Í1 – ˙
102. B.E. = 180.69 kJ/mole fi w = hv0 l Î 4˚
180.69 6.62 ¥ 10 –34 ¥ 3 ¥ 108 3
eV/atom = hv0 = 2.17 ¥ 10 –18 ¥
96.368 l 4
180.69
¥ 1.6 ¥ 10-19 = 6.6 ¥ 10-34 ¥ v0 4 ¥ 6.62 ¥ 3 ¥ 10 –26
96.368 l= = 12.20 ¥ 10 –8 m = 1220 Å
6.51 ¥ 10 –18
v0 = 6.626 ¥ 10-34
2.17 ¥ 10 –18 2.17 ¥ 10 –18
103. hn1 = hn0 + 2E1 and hn2 = hn0 + E1 10. DE = 2
¥ Z2 = = 5.42 ¥ 10-19
n 4
hn1 – hn0 = 2E1 and hn2 – w0 = E1 or 5.42 ¥ 10–12 erg
hn - w0 hc
2= 1 \ 2hn 2 - 2 w0 = hn1 - w0  DE =
hn 2 - w0 l
h[2n2 – n1] = w0 6.626 ¥ 10 –34 ¥ 3 ¥ 108
l= = 3.67 ¥ 10-7 meter
w0 = 6.62 ¥ 10–34 (4 ¥ 1015 – 3.2 ¥ 1015) 5.42 ¥ 10 –19
w0 = 6.62 ¥ 10–34 ¥ 0.8 ¥ 1015 or 3.67 ¥ 10–5 cm
w0 = 5.29 ¥ 10–19 w0 = 318.9 kJ/mol 11. For n = 3 and l = 2 (i.e., 3d orbital), the values of m
varies from –2 to +2, i.e., –2, –1, 0, +2 and for each ‘m’
there are 2 values of ‘s’, i.e., +1/2 and –1/2.
PREVIOUS YEARS’ QUESTIONS Maximum number of electrons in all the five d-orbitals
OF JEE (MAIN & ADVANCED) is 10.
2 1 1 – 21.76 ¥ 10 –19 ¥ 4
1. (d) Charge/mass for n = 0, a = , p = , e = 12. E rd = J
4 1 1/1837 3 9
So, order is n, a, p, e 87.04
2. (a) Isotonic : same number of neutrons = ¥ 10 –19 = 9.67 ¥ 10 –19 J
9
3. (c) Conceptual
hc
h 6.6 ¥ 10-34 \ DE =
4. (c) l = = = 2.37 ¥ 10-30 m l
mv (0.2) ¥ 5/(60 ¥ 60)
6.626 ¥ 10 –34 ¥ 3 ¥ 108
5. (a) Z = 37, so electronic configuration is 1s2 2s2 2p 3s2 l=
3p6 4s2 3s10 4p6 5s1 9.67 ¥ 10 –19
\ for outermost electron = 2.0556 ¥ 10–7 M = 2055 Å
6. (c) Most electro-negative element is F. h 6.62 ¥ 10 –34
\ electronic configuration: ns2np5 13. l = =
mu 0.1 ¥ 100
7. Average mass = Mass of isotopes ¥ Mole fraction +
Mass of isotopes ¥ Mole fraction or l = 6.62 ¥ 10–35 m = 6.62 ¥ 10–25 Å
10.81 = 11.01 ¥ x + 10.01 (1 – x) 14. For hydrogen atom, Z = 1, n = 1
10.81 = 11.01 x + 10.01 – 10.01 x Z
V = 2.18 ¥ 106 ¥ ms –1 = 2.18 ¥ 106 ms–1
x = 10.81 – 10.01 = 0.80 n
% = 0.80 ¥ 100 = 80% de Broglie wavelength,

IIT JEE PC-V1_02.indd 46 3/21/2017 5:16:06 PM


Join our telegram channel: @Jee_neet_topper

Atomic Structure 2.47

Kinetic energy = Absorbed energy – Bond energy


h 6.26 ¥ 10-34
l= = \ kinetic energy = 4.417 ¥ 10–19 – 3.984 ¥ 10–19 J
mv 9.1 ¥ 10 –31 ¥ 2.18 ¥ 106
= 3.34 ¥ 10–10 m = 3.3 Å = 4.33 ¥ 10–20 J
\ kinetic energy of each atom
For 2p, l = 1
h 4.33 ¥ 10 –20
\ Orbital angular momentum 2 = = 2.165 ¥ 10–20 J
hc 2p 2
15. From DE = Ef – Ei = 19. The shortest wavelength transition in the Balmer series
l
corresponds to the transition
È 1 1 ˘
2.18 ¥ 10–18 ¥ Z2 Í 2 – n = 2 Æ n = •. Hence, n1 = 2, n2 = •
Î (1) (2) 2 ˙˚
Ê 1 1ˆ Ê 1 1 ˆ
6.626 ¥ 10 –34 ¥ 3 ¥ 108 n = RH Á 2 – 2 ˜ = (109677 cm –1 ) ¥ Á 2 – 2 ˜
= Ë n1 n2 ¯ Ë 2 • ¯
3 ¥ 10 –8
= 27419.25 cm -1
3
or –2.18 ¥ 10–18 ¥ Z2 ¥ = 6.626 ¥ 10–18 20. As the a particle travelling with velocity, ‘u’ stops at a
4
6.626 ¥ 10 –18
¥ 4 distance 10–13 m, its K.E. becomes zero and gets con-
Z2 = ª4 verted in to P.E.
2.18 ¥ 10 –18 ¥ 3
Z=2 1 2 1 2 Ze 2 Ze 2
mu = ¥ or u 2 =
for He+ Z = 2 2 4pe 0 r pe 0 ◊ m.r
16. DEHe+ = DEH
Here, Z = 29 for Cu atom
È1 1 ˘ 29 ¥ (1.6 ¥ 10 –19 ) 2
or -2.18 ¥ 10-18 ¥ Z 2 Í – ˙ fi u2 =
Î 4 16 ˚
3.14 ¥ 8.85 ¥ 10 –12
È1 1˘
= – 2.18 ¥ 10 –18 Í 2 – 2 ˙ ¥ (4 ¥ 1.672 ¥ 10 –27 ) ¥ 10 –13
Î n1 n2 ˚
\ u = 6.3 ¥ 106 m sec–1
È1 1 ˘ È 1 1˘
4Í - ˙ = Í 2 - 2 ˙ 1 2 h
Î 4 16 ˚ Î n1 n2 ˚ 21. mu = eV; also l =
2 mu
\ n1 = 1 and n2 = 2
17. rn for H = r1 ¥ n2 1 h2 1 h2
\ m 2 2 eV or V =
R3 for H = 0.529 ¥ 9 ¥ 10–8 cm = 529 ¥ 9 ¥ 10–10 m 2 m l 2 ml 2e
( r1 = 0.529 A°)
1 ¥ (6.62 ¥ 10 –34 ) 2
u V=
Also un = Z 1 2 ¥ 9.108 ¥ 10 –31 ¥ (1.54 ¥ 10 –10 ) 2 ¥ 1.602 ¥ 10 –19
n
= 63.3 volt
2.19 ¥ 108 2.19 ¥ 106
\ u3 = cm sec –1 msec –1 22. Determination of number of moles of hydrogen gas,
3 3
( u1 = 2.19 ¥ 108 cm sec–1) PV 1¥1
n= = = 0.0409
\ Number of waves in one round RT 0.082 ¥ 298
2p r3 2p r3 2p r3 ¥ u3 ¥ m The concerned reaction is H2 Æ 2H; DH = 436 kJ mol–1.
= = = Energy required to bring 0.0409 moles of hydrogen gas
l h /mu3 h
to atomic state = 436 ¥ 0.0409 = 17.83 kJ.
2 ¥ 22 ¥ 0.529 ¥ 9 ¥ 10 –10 ¥ 2.19 Calculation of total number of hydrogen atoms in
¥ 106 ¥ 9.108 ¥ 10 –31 0.0409 mole of H2 gas.
= =3 1 mole of H2 gas has 6.02 ¥ 1023 molecules
7 ¥ 3 ¥ 6.62 ¥ 10 –34
6.02 ¥ 1023
18. Bond energy of I2 =240 kJ mol–1 = 240 ¥103 J mol–1 0.0409 mole of H2 gas = ¥ 0.0409
1
240 ¥ 103
= J mol –1 = 3.984 ¥ 10–19 J mol–1 Since 1 molecule of H2 gas has 2 hydrogen atoms, 6.02
6.023 ¥ 1023 ¥ 1023 ¥ 0.0409 molecules of H2 gas = 2 ¥ 6.02 ¥ 1023
hc 6.626 ¥ 10 –34 Js ¥ 3 ¥ 108 ms –1 ¥ 0.0409 = 4.92 ¥ 1022 atoms.
Energy absorbed = =
l 4500 ¥ 10 –10 m Energy required to excited an electron from the ground
= 4.417 ¥ 10–19 J state to the next excited state

IIT JEE PC-V1_02.indd 47 3/21/2017 5:16:08 PM


Join our telegram channel: @Jee_neet_topper

2.48 Physical Chemistry-I for JEE (Main & Advanced)

Ê 1 (b) To describe a one electron wave function, three


1ˆ Ê1 1ˆ
= 13.6 Á 2 – 2 ˜ eV = 13.6 ¥ Á – ˜ quantum numbers n, l and m are needed. Further
Ë n1 n2 ¯ Ë 1 4¯
to abide by Pauli exclusion principle, spin quan-
3 1.6 ¥ 10-19 tum number (s) is also needed.
= 13.6 ¥= 10.2 eV = 10.2 ¥ KJ
4 1000 (c) For shape, size and orientation, only n, l and m are
Therefore, energy required to excite 4.92 ¥ 1022 elec- needed.
trons (d) Probability density (y2) can be determined if n, l
= 1.632 ¥ 10–21 4.92 ¥ 1022 kJ and m are known.
= 8.03 ¥ 10 = 80.3 kJ
Therefore, total energy required = 17.83 + 80.3 Comprehension-Based Questions
= 98.17 kJ 27. fi n = 2, i.e., S1 = 2s orbital.
23. For maximum energy, n1 = 1 and n2 = • 28. Ground state energy of electron in H –atom.(EH)
1 Ê 1 1ˆ
= RH Z 2 Á 2 – 2 ˜ [3]2
l Ë n1 n2 ¯ -13.6 ev
( ELi+2 ) S1 [2]2 9
= = = 2.25
Since RH is a constant and transition remains the same EH [1] 2
4
-13.6 ¥ 2 ev
1 l Z2 1 [1]
µ Z 2 ; He = 2H =
l lH Z He 4
29. In S2 state, E (Li2+) = k (given)
1
Hence, lHe = ¥ 91.2 = 22.8 nm (3) 2 (1) 2
4 ELi+2 = -13.6 ev = 13.6 ¥ ev
2 n22 (1) 2
24. y 2s = Probability of finding electron within 2s sphere
2 \ n2 = 3 = S2
y 2s = 0 (at node)
( probability of finding an electron is zero at node) Integer-Type Questions
2 2
È 1 1 Ê 1 ˆ 3/2 Ê r0 ˆ ˘ 30. When n = 3, l = 0, 1, 2, i.e., there are 3s, 3p and 3d
\ 0=Í Á ˜ Á 2 - ˜ ¥ e – r0 / a0 ˙ orbitals. If all these orbitals are completely occupied as
ÍÎ 4 2p Ë a0 ¯ Ë a0 ¯ ˙˚
È r0 ˘ r0
fi Í2 – a ˙ = 0; \ 2 = a ; 2a0 = r0
Î 0˚ 0 1
25. A Æ r; B Æ q, C Æ p, D Æ s Total 18 electrons, 9 electrons with s = + and 9 with
2
1
(a) Potential Energy s=- .
2
1 Ê Ze 2 ˆ z2
= - ÁË ˜¯ = -13.6 ¥ 2 2 ev 13.6
4pe 0 r n 31. \ En = - eV where, n = 1, 2, 3..........
n2
z2 -13.6
Kinetic energy = +13.6 ev In excited states, E2 = = - 3.4 eV
n2 4
32. (a) Given, atomic number of Rb, Z = 37
1 Ê Ze 2 ˆ Vn
Kn = ÁË ˜¯ fi = - 2( R ) Thus, its electronic configuration is [Kr]5s1. Since, the
8pe 0 r Kn last electron or valence electron enter is 5s subshell.
Ze 2 Z2 So, the quantum numbers, are n = 5, l = 0, (for s-orbit-
(b) En = - = -13.6 2 ev µ r -1 fi x = -1 al) m = 0  m = + l to – l, s = +1/2 or –1/2.
8pe 0 r n
h ÈZ2 ˘
(c) Angular momentum = l (l + 1) 33. (a) Given, in the question E = - 2.178 ¥ 10-18 J Í 2 ˙
2p În ˚
= 0 in S orbital For hydrogen Z = 1,
2 2
an n È1˘
(d) rn = 0 = 0.529 Å So, E1 = - 2.178 ¥ 10-18 J Í 2 ˙
z z În ˚
26. A Æ q; B Æ p, q, r, s; C Æ p, q, r ; D Æ p, q, r
È1˘
h E2 = - 2.178 ¥ 10-18 J Í 2 ˙
(a) Orbital angular momentum (L) = l + (l + 1) Î2 ˚
2p
i.e., depends on azimuthal quantum number only. Now, E1 – E2 i.e.

IIT JEE PC-V1_02.indd 48 3/21/2017 5:16:10 PM


Join our telegram channel: @Jee_neet_topper

Atomic Structure 2.49

1 ˆ hc 4s 4p 4d 4d
Ê1
DE = 2.178 ¥ 10-18 Á 2 - 2 ˜ = 0 –1 0 +1 –2 –1 0 +1 +2 –3 –2 –1 0 +1 +2 +3
Ë1 2 ¯ l
1 ˆ 6.62 ¥ 10-34 ¥ 3.0 ¥ 108 According to question |m1| = 1, i.e. there are two pos-
Ê1
2.178 ¥ 10-18 Á 2 - 2 ˜ = sible values of m1, i.e. +1 and –1 and one orbital can
Ë1 2 ¯ l
1
\ l = 1.21 ¥ 10–7 m contain maximum two electrons one having s = +
and other having s = –1/2. 2
34. (c) According to Bohr’s model,
So, total number of orbitals having {|m1| = 1} = 6
nh n2h2 Total number of electrons having
mvr = fi (mv) 2 = 2 2
2p 4p r 1¸
Ï
1 2 n2h2 Ì| m1 | = 1 and m2 = ˝ = 6
fi Kinetc Energy = mv = 2 2 Ó 2˛
2 8p r m 1 3
Also, Bohr’s radius for H-atom is, r = n2a0 36. (5) Hint: K.E. = MV 2 = RT
2 2
Substituting ‘r’ in Eq. (i) gives
\ m 2v 2 = 2mKE \ mv = 2mKE
h2 h2
K.E. = 2 2 2 when n = 2, K.E. = h h h
8p n a0 m 32p 2 a02 m l(wavelength) = = µ
mv
2mKE 2m(T )
35. (6) Hint: This problem is based on correct of quantum
number. Follow the following steps to solve this prob- Where, T = Temperature in Kelvin
lem. Write all possible orbital's having combination of h
l (He at – 73°C = 200 K) =
same principal, azimuthal, magnetic and spin quantum 2 ¥ 20 ¥ 1000
number. Then count the all possible electrons having
given set of quantum numbers. For n = 4, the total num- l (He) 2 ¥ 20 ¥ 1000
\ =M = =5
ber of possible orbitals are l (Ne) 2 ¥ 4 ¥ 200
Thus, M = 5

IIT JEE PC-V1_02.indd 49 3/21/2017 5:16:10 PM

You might also like